Jump to content

Wikipedia:Reference desk/Humanities: Difference between revisions

From Wikipedia, the free encyclopedia
Content deleted Content added
Line 727: Line 727:
:I think you are talking about [[Thomas Babington Macaulay]]. I don't think he was mute before, but on having had hot coffee spilt on him at the age of three, is said to have announced: "Thank you, Madam, the agony is sensibly abated". The story is recounted [http://www.historytoday.com/richard-cavendish/birth-thomas-babington-macaulay here], but I observe it is not in our article or in [[:wikiquote:Thomas Babington Macauley]]. --[[User:ColinFine|ColinFine]] ([[User talk:ColinFine|talk]]) 09:26, 27 July 2015 (UTC)
:I think you are talking about [[Thomas Babington Macaulay]]. I don't think he was mute before, but on having had hot coffee spilt on him at the age of three, is said to have announced: "Thank you, Madam, the agony is sensibly abated". The story is recounted [http://www.historytoday.com/richard-cavendish/birth-thomas-babington-macaulay here], but I observe it is not in our article or in [[:wikiquote:Thomas Babington Macauley]]. --[[User:ColinFine|ColinFine]] ([[User talk:ColinFine|talk]]) 09:26, 27 July 2015 (UTC)
::That looks right, thanks. Wonder where I got the mute bit from? --[[User:Dweller|Dweller]] ([[User talk:Dweller|talk]]) 09:32, 27 July 2015 (UTC)
::That looks right, thanks. Wonder where I got the mute bit from? --[[User:Dweller|Dweller]] ([[User talk:Dweller|talk]]) 09:32, 27 July 2015 (UTC)
:::In the version I read, he had not previously spoken up to that point; there may have been confusion in variant accounts between "mute" meaning "not speaking" and "mute" meaning "unable to speak." {The poster formerly known as 87.81.230.195} [[Special:Contributions/212.95.237.92|212.95.237.92]] ([[User talk:212.95.237.92|talk]]) 13:06, 27 July 2015 (UTC)


== Deleuze & Guattari ==
== Deleuze & Guattari ==

Revision as of 13:06, 27 July 2015

Welcome to the humanities section
of the Wikipedia reference desk.
Select a section:
Want a faster answer?

Main page: Help searching Wikipedia

   

How can I get my question answered?

  • Select the section of the desk that best fits the general topic of your question (see the navigation column to the right).
  • Post your question to only one section, providing a short header that gives the topic of your question.
  • Type '~~~~' (that is, four tilde characters) at the end – this signs and dates your contribution so we know who wrote what and when.
  • Don't post personal contact information – it will be removed. Any answers will be provided here.
  • Please be as specific as possible, and include all relevant context – the usefulness of answers may depend on the context.
  • Note:
    • We don't answer (and may remove) questions that require medical diagnosis or legal advice.
    • We don't answer requests for opinions, predictions or debate.
    • We don't do your homework for you, though we'll help you past the stuck point.
    • We don't conduct original research or provide a free source of ideas, but we'll help you find information you need.



How do I answer a question?

Main page: Wikipedia:Reference desk/Guidelines

  • The best answers address the question directly, and back up facts with wikilinks and links to sources. Do not edit others' comments and do not give any medical or legal advice.
See also:


July 20

Why did Bush and Obama let North Korea get nuclear bombs but not Iran?

Did they have intelligence that the craziness is all bullshit and the Iranians' craziness is real? Did North Korea make too many dumb empty threats? I guess in their favor the Koreans just want to have something intact enough to invade and the Iranians want to commit genocide and country destruction. Sagittarian Milky Way (talk) 01:52, 20 July 2015 (UTC)[reply]

Iran is closer to places that have lots of oil. ;) --Jeffro77 (talk) 01:54, 20 July 2015 (UTC)[reply]
Tthe U.S. is clearly in less of a position to influence what goes on in North Korea. The Chinese government considers it very much within their sphere of influence, and while they may not like the idea of North Korea having nuclear weapons, they probably like the idea of the U.S. deciding the issue even less. As for the suggestion that Iran wants to engage in 'genocide and country destruction', I very much doubt it - their regional ambitions have much more to do with their rivalry with Saudi Arabia as the local head honcho, and they know full well that any use of nuclear weapons would be suicidal. Don't mistake bellicose rhetoric for military ambition. AndyTheGrump (talk) 02:07, 20 July 2015 (UTC)[reply]
Hitler and Japan should've known that starting WWII that early was suicidal. But no need to copy, if Iran's leaders are rational then they are what they are. They must fantasize though, right? If they had advanced nuclear tech and the rest of the world had none?.. Maybe they'd bomb Saudi Arabia, too? Or maybe the people at the top don't actually believe Allah and just want power? So Israel's not that important then. Sagittarian Milky Way (talk) 02:57, 20 July 2015 (UTC)[reply]
  • I'm not sure it has much to do with let. Iran depends on foreign trade much more than North Korea, thus the regime of Sanctions against Iran are a reasonable tool to use to put pressure on Iran to curtail their nuclear weapons program. North Korea is quite isolationist, and has little in the way of raw materials or finished goods it trades with the outside world anyways, it's hard to hold the sanction of "we'll cut off trade with you" with a country that doesn't trade much with the outside world anyways. --Jayron32 04:28, 20 July 2015 (UTC)[reply]
  • Yeah... The US did not "let" North Korea get the bomb... The US did everything they could think of (short of going to war) in an effort to stop the Koreans from getting nukes. North Korea developed the bomb despite everything the US tried to do. The UN imposed extremely harsh sanctions on North Korea (even harsher than the ones imposed on Iran). The North Korean government simply ignored these sanctions, and continued on until they had a bomb. The North Korean government was willing to let its people suffer great hardship in order to get the bomb. It is possible that, had the recent talks failed, the Iranian government might have decided to do the same... continue development despite continued sanctions. Whether the Iranian people would have been willing to suffer as much as the Korean people did is another question (and one that we will never know the answer to). Blueboar (talk) 01:54, 21 July 2015 (UTC)[reply]

Country With The Highest Life Expectancy In 1950

What was the country with the highest life expectancy in 1950? What was its life expectancy?

125.255.167.126 (talk) 05:07, 20 July 2015 (UTC)[reply]

According to http://ourworldindata.org/wp-content/uploads/datamaps/LifeExpectancyWorldMaps/Life_Expectancy_WorldMap_1950.html it seems Norway topped the list with a life expectancy (at birth) of 71.6 years. Gabbe (talk) 07:46, 20 July 2015 (UTC)[reply]
According to the World Bank, the highest life expectancy is Japan. See http://data.worldbank.org/indicator/SP.DYN.LE00.IN/countries/1W?page=3&display=default
Both sex
Country Name 1995
  1. Japan 79.5
  2. Sweden 78.7
  3. Hong Kong 78.7
  4. Switzerland 78.4
  5. Italy 78.2
  6. Iceland 78.0
  7. Spain 78.0
  8. Canada 78.0
  9. Australia77.8
  10. France 77.8
―― Phoenix7777 (talk) 23:53, 20 July 2015 (UTC)[reply]
...but the question was for 1950, not 1995. --Stephan Schulz (talk) 16:00, 22 July 2015 (UTC)[reply]
Three out of four digits match, whattaya want for nothing? —Tamfang (talk) 19:09, 22 July 2015 (UTC)[reply]

World Infant Mortality Rate

What was the infant mortality rate of the world in 1850, 1900, 1950 and 2000?

Nineguy (talk) 05:12, 20 July 2015 (UTC)[reply]

You want the average infant mortality rate for the whole world or the median with upper end and lower end? The answer rests on that knowledge, because there is a disparity in infant mortality rate between wealthier, developed nations and impoverished, undeveloped nations. 71.79.234.132 (talk) 14:34, 20 July 2015 (UTC)[reply]

See http://data.worldbank.org/indicator/SP.DYN.LE00.IN/countries/1W?page=3&display=default―― Phoenix7777 (talk) 00:20, 21 July 2015 (UTC)[reply]
Also the very definition of "infant mortality" is nicely inconsistent across the world. Collect (talk) 18:42, 24 July 2015 (UTC)[reply]

Why did the UK lose economic ground to France, Germany and Italy?

As I understand it, in about 1960, the UK was richer than the other three countries, as measured by GDP per capita. But, over time, these countries overtook the UK. Why did this happen?--Leon (talk) 20:43, 20 July 2015 (UTC)[reply]

See Economic history of the United Kingdom. In two words - Harold Wilson. In a few more words - NEDC, Department of Economic Affairs, Ministry of Technology, decimalization, British Leyland... Tevildo (talk) 22:09, 20 July 2015 (UTC)[reply]
The basic reason is a case of the common but absurd obsession with foreign currency exchange rates, leading to excessive high interest rates to "protect" an exchange rate and the highly destructive consequences of high interest. The greatest damage was probably under Thatcher. Trying to remember a good paper on this. When George Soros broke the pound later, he made himself a fortune but helped the UK far more by helping it change from a destructively following this traditional obsession. Of course, the UK was helped more recently keeping the pound, not going into the Euro death trap, like those other 3.John Z (talk) 22:25, 20 July 2015 (UTC)[reply]
You have probably made a mistake, although the broad principles are correct. The 1967 devaluation (Harold Wilson) was forced by market pressures on sterling; one of Mrs. Thatcher's first acts had been to do away with currency controls - those of us who lived through the '70s can remember the absurdity of trying to go on foreign holiday with limited currency. Mrs. Thatcher actually sorted out the In Place of Strife mess - vide the 1981 budget and the Trade Union legislation (1980, 1982, and 1984). Nigel Lawson, in the late 1980s, adopted a policy of shadowing the Deutschmark, which predictably failed. The main policy failure was not under Thatcher but the hopeless incompetent John Major and Black Wednesday. --95.49.76.116 (talk) 08:50, 22 July 2015 (UTC)[reply]
I'm sure you're right about the details- not British. Thatcher did keep the UK out of the Euro death-trap - every nation in Europe's economy would have been better off if not for this worst designed currency of all time. Cf. Wynne Godley's denunciations & predictions. But high interest and high unemployment under Thatcher was probably the biggest single long-term damage the UK economy saw postwar - but a bit different from the very long term broad trend though. Geoffrey Gardiner, relatively sympathetically, calls her policies a curate's egg. Again, as critical as I am of Thatcher (from a (post-)Keynesian/ Institutional/ MMT perspective) the damage wasn't as bad as entering the Euro would have been.John Z (talk) 23:03, 23 July 2015 (UTC)[reply]
Again, I am sorry to disagree; there was nothing sensible about continuing to subsidise the production of cars that no-one wanted to buy (and were a byword for mediocrity), and over-priced ships, steel, and coal. It was certainly harsh on the workforces involved that these industries were rapidly restructured; those that were capable survived - British Gas, British Telecom, and even (for a while), British Leyland - and those that weren't closed down, as the British economy was restructured from a manufacturing economy to a service economy. That was undoubtedly the correct course; the high interest rates and high unemployment, which were a feature of Thatcher's first term, arose out of the previous policy failures of the Callaghan government - high inflation and Trade Union enforced subsidy of uneconomic industries - all of which came to a head in the Winter of Discontent. --31.63.162.53 (talk) 15:35, 26 July 2015 (UTC)[reply]
Germany seems to have done quite well out of that supposed death trap. Also despite (or maybe because?) not being in the Euro, Britain seems to have struggled more than the others in the financial crisis. 86.189.248.177 (talk) 23:15, 21 July 2015 (UTC)[reply]
That is because, just as the Euro is overvalued to the Greeks, it is undervalued to the Germans. If we still had the Drachma and the Deutschmark, the Drachma would be worth half as much as the Euro and the Deutschmark twice as much. In other words, the very collapse of the Greek economy (and the Italian, Spanish, Portuguese, and Irish economies) is pulling the value of the Euro down and making it easier for German exporters to sell their goods; if either the Germans or the Greeks were outside the Euro, the price of your Mercedes car would be prohibitively expensive. Not that the Germans see it that way, of course. 95.49.76.116 (talk) 08:56, 22 July 2015 (UTC)[reply]
There's also the issue of the productivity of British workers, which is still a problem; see UK productivity gap with developed nations now widest for 20 years (2014) and Average UK workers pose a productivity puzzle (2015). Alansplodge (talk) 10:28, 21 July 2015 (UTC)[reply]

20s vs 40s

I;m a 27 year old guy but am pondering whether to propose to a 40 year old. I suspect she will accept but our families probably not, seeing our age difference as weird. I was wondering how uncommon it is for a man to marry a 40 year old woman. Any stats? 78.144.241.217 (talk) 21:11, 20 July 2015 (UTC)[reply]

See Age disparity#Statistics. -- ToE 21:19, 20 July 2015 (UTC)[reply]
See also Age disparity in sexual relationships. ←Baseball Bugs What's up, Doc? carrots21:20, 20 July 2015 (UTC)[reply]
Aha, same article. ←Baseball Bugs What's up, Doc? carrots21:20, 20 July 2015 (UTC)[reply]
This [1] article from the Chicago Tribune gives some statistics and links to further studies. Relevant to your case - "A 2003 AARP study found 34 percent of women older than 40 were dating younger men, with 8 percent seeing men 10 or more years younger." So your situation is not the most common, but it's not that rare either. (Bad Religion, Lady Gaga and OK Go all agree on the matter - "Do What You Want" [2] [3] [4] :) SemanticMantis (talk) 21:41, 20 July 2015 (UTC)[reply]
Rita Rudner is about 4 years older than her husband. She said, "The old theory was, marry an older man, they're more mature. The new theory is, men don't mature, so marry a younger one." ←Baseball Bugs What's up, Doc? carrots22:21, 20 July 2015 (UTC)[reply]

July 21

Did anyone die from heat on 8 July 2003 in Dhahran?

On 8 July 2003 Dhahran experienced the highest dew point since we have been recording them. It had a 95F dew point and a temperature of 108F. This gave the city a heat index of 174F. Combined with a 3 MPH wind, this must have been awful. I have been able to find all sorts of stats on the weather for that day, but I haven't been able to find out if anybody died. Does anyone here have access to newspapers from the area or some other way to determine the death count (if there is any).

Sources: Ask Tom why: What is the highest dew point ever recorded?; Extreme Weather and Climate By C. Donald Ahrens, Perry Samson; Weather History for OEDR Tuesday, July 8, 2003

Zell Faze (talk) 00:27, 21 July 2015 (UTC)[reply]

Absent someone turning up a real archive, you could use google to search the websites of Saudi newspapers on that date and shortly thereafter. For example, here are the results for the English-language Arab News for 9 July 2003 (their date format) [5].
One other note - even if the heat that day did kill, this fact might not be recorded. [6] and [7] say many deaths due to heat aren't identified as such; that it's only after some time has passed and authorities notice the average death rate is elevated that the heat can be blamed. Sorry I haven't been able to find if the Dhahran record was part of a heatwave. 184.147.131.217 (talk) 12:07, 21 July 2015 (UTC)[reply]

Living to age 100 or more

Are there any data or statistics that would answer a question such as this: "A person born in the year _____ has a _____ percent chance of living to age 100 or more"? Thanks. Joseph A. Spadaro (talk) 03:22, 21 July 2015 (UTC)[reply]

The first link when I google "likelihood of living one hundred" is this: "How likely are you to live to 100? Get the full data", which seems to be estimates for people born in the United Kingdom over the last one hundred years. Gabbe (talk) 04:48, 21 July 2015 (UTC)[reply]
The general concept you're looking for is an actuarial table. -Elmer Clark (talk) 10:34, 21 July 2015 (UTC)[reply]
Thanks. It is safe to assume that statistics for the UK would be somewhat similar to that of the USA? Or not, for some reason? Joseph A. Spadaro (talk) 18:01, 21 July 2015 (UTC)[reply]
This https://www.wolframalpha.com/input/?i=US+life+expectancy+20+year+old Wolfram|Alpha query will get you going. In general, the US life expectancy is slightly lower than that for Western Europe. You can also add "male" or "female" after the country, if you are interested in a specific gender. LongHairedFop (talk) 18:15, 22 July 2015 (UTC)[reply]
Very interesting website. Thanks. Joseph A. Spadaro (talk) 02:53, 23 July 2015 (UTC)[reply]

Thanks, all. Joseph A. Spadaro (talk) 18:33, 25 July 2015 (UTC)[reply]

to find correct reference Mkalburge (talk) 06:36, 21 July 2015 (UTC)

Hello,

I read about Francois Bernier on your website. URL - https://en.wikipedia.org/wiki/Fran%C3%A7ois_Bernier

On that page a paper written by Bernier is mentioned and that is 'A New Division of the Earth'.

In the References section it is given that 'François Bernier, "A New Division of the Earth", in Journal des sçavans (April 24, 1684). Translated by T. Bendyphe in "Memoirs Read Before the Anthropological Society of London" Vol 1, 1863–64, pp 360–64.'

I found the 1st volume of the 'Memoirs Read Before the Anthropological Society of London', but in it could not find the paper.

Can you please help me to find this paper? Where can I find it ? It is important.

Please reply.

@User:Mkalburge
Bernier, François (2001-04-01). "A New Division of the Earth". History Workshop Journal (51): 247–250. ISSN 1363-3554. JSTOR 4289731. Retrieved 2015-07-21.
"Register & Read". JSTOR.. Free individual registration, offering free read-only access (no printing or saving) to three articles every two weeks (seventy-eight per year).
Your local public or academic library may also offer free institutional access to JSTOR.
If you've tried both of the above and they don't work for you, try re-posting to Resource Request
I've added the source as an external link to the François_Bernier article. Please use it as a reference if you can improve the article.
-- Paulscrawl (talk) 07:07, 21 July 2015 (UTC)[reply]
P.S. - sorry for delay; got distracted with another author's article, "On the Phallic Worship of India", in source book found on the Internet Archive - here is the direct link to page 360:
Bernier, François (1864). "A New Division of the Earth". Memoirs Read Before the Anthropological Society of London. Vol 1. 1863–64. London: Trübner and Co. pp. 360–364. Retrieved 21 July 2015.
Now hyperlinked in article References. Please use to improve article - and welcome to Wikipedia!
-- Paulscrawl (talk) 08:30, 21 July 2015 (UTC)[reply]

dames & knights

my query is in regard to British royal titles. When a man is knighted he is thereafter known as Sir .... and his wife is know as Lady..... When a woman is given a Damehood, what title, if any, is given to her husband? — Preceding unsigned comment added by Janinsane (talk contribs) 10:04, 21 July 2015 (UTC)[reply]

None.
121.211.12.111 (talk) 10:11, 21 July 2015 (UTC)[reply]
Agreed - see TITLES AND STYLES OF KNIGHTS AND DAMES. An attempt in 2012 to address this issue and that of same-sex partners (Civil partners of knights and peers should get honorary title just like wives do, MP argues) seems to have come to nothing. Reform can be a slow business in the UK. Alansplodge (talk) 10:19, 21 July 2015 (UTC)[reply]
Can the British parliament change the New Zealand rulebook like that? Or would that only fly in their realm? InedibleHulk (talk) 05:23, July 22, 2015 (UTC)

Females on slave ships.

Were females on trans-atlantic slave ships wearing shackles and manacles like the men? Secondly, how did they look after their children on the slave ships? --Constiniolp (talk) 14:30, 21 July 2015 (UTC)[reply]

According to this, "Women and children were kept in separate quarters, sometimes on deck, allowing them limited freedom of movement, but this also exposed them to violence and sexual abuse from the crew." AndrewWTaylor (talk) 16:26, 21 July 2015 (UTC)[reply]

Two sources most any library will likely have:

All the European nations lodged the two sexes of their slaves apart, as usual following Portuguese practice, ordinarily “by means of a strong partition at the main mast, the forepart is for men, the other behind the mast for the women. If it be in large ships carrying five or six hundred slaves, the deck in such ships ought to be at least five and a half or six feet high [being] the more airy and convenient for such a considerable number of human creatures; and consequently far the more healthy for them.” Female slaves were treated better than the men, not being chained. The reason for these arrangements was not only to prevent the male slaves from seducing the women but also that black women were often said to do what they could to urge the men to assert themselves and attack the crew.

Thomas, Hugh (1997). The Slave Trade: The Story of the Atlantic Slave Trade, 1440 - 1870. New York, NY: Simon & Schuster. ISBN 978-0-684-81063-8. His quoted source in first sentence is (apparently, not clearly noted): Elizabeth Donnan, Documents Illustrative of the Slave Trade to America, 4 vols., Washington,DC: Carnegie Institute, 1935, vol. I, p. 272. All four volumes of primary source material online here

The Middle Passage was exceptionally hard for women. They were often kept on deck and at proximity to the ship crew, who would routinely rape and abuse them. The crew would often make them sing and dance as a mode of humiliation and entertainment. This was a practice called ‘‘dancing the slaves,’’ which occurred regularly during the passage. The suicide rates during the Middle Passage were higher for enslaved women, because they chose to jump ship rather than endure the horrors of slavery.

Deetz, Kelley (2007). "Women". In Falola, Toyin; Warnock, Amanda (eds.). Encyclopedia of the Middle Passage. Westport, Connecticut: Greenwood Press. p. 405. ISBN 978-0-313-33480-1.. Other articles listed in the analytic table of contents under "Middle Passage Experience" include "Children", "Families and Family Separations", "Rape and Sexual Abuse", and others relevant to your questions. Many public and academic libraries offer online access to the e-book edition.

Paulscrawl (talk) 20:00, 21 July 2015 (UTC)[reply]

English-speaking Buddhists

How do English-speaking Buddhists meditate on the Buddhist prayer beads (aka Buddhist rosary)? Do they chant in, for example, the name of the Amitabha Buddha in a non-English language or use an English translation? 140.254.226.190 (talk) 14:40, 21 July 2015 (UTC)[reply]

Most I've encountered or read (who bother with mantras) use the language most closely associated with the tradition they belong to, or just Sanskrit.
(There are some English-speakers who call themselves Buddhists, who really mean they're just atheists who don't want to appear anti-religious. Some of them will admit this, the ones that won't will still try to get out of saying mantras.) Ian.thomson (talk) 15:17, 21 July 2015 (UTC)[reply]
How strange. Anglophones say the Roman Catholic rosary in English. Everything is recited too straight from memory. I think they use the vernacular language, because they want to understand what they are saying. I wonder if the atheists that you met would bother with saying the Catholic rosary, which is explicitly theistic, but they chant in Latin so they don't even know what they're saying. 140.254.226.190 (talk) 15:39, 21 July 2015 (UTC)[reply]
Catholics use the vernacular since the Second Vatican Council, but it used to be common for many Catholics to use Latin (or what they could make of it) -- see Patter. Ian.thomson (talk) 16:01, 21 July 2015 (UTC)[reply]

Why can't celibate Catholic priests castrate themselves?

What is the justification against castration and becoming an eunuch? I read it on the clerical celibacy page. 140.254.226.190 (talk) 14:50, 21 July 2015 (UTC)[reply]

See Castration#Christianity: the First Council of Nicea forbade voluntary castration except for health reasons, probably because most surgery just wasn't safe until the late 1800s. Ian.thomson (talk) 15:11, 21 July 2015 (UTC)[reply]
But since surgery is relatively safe now, why can't eunuchs become priests? 140.254.226.190 (talk) 15:16, 21 July 2015 (UTC)[reply]
I doubt safety was the only concern for the bishops at Nicea, but in any case, according to Roman Catholic dogma, the decisions of the Ecumenical councils are infallible, and therefore it is not possible to revise them. - Lindert (talk) 15:32, 21 July 2015 (UTC)[reply]
Btw, eunuchs can become priests if they were castrated against their will. One argument against willful castration by the church father John Chrysostom is that it is an insult to God's creation, because it implies that the sin is caused by our bodies, rather than our personal choices: "For to cut off our members has been from the beginning a work of demoniacal agency, and satanic device, that they may bring up a bad report upon the work of God, that they may mar this living creature, that imputing all not to the choice, but to the nature of our members, the more part of them may sin in security, as being irresponsible; and doubly harm this living creature, both by mutilating the members, and by impeding the forwardness of the free choice in behalf of good deeds." (source). - Lindert (talk) 15:49, 21 July 2015 (UTC)[reply]
I thought Testicles was a Greek philosopher. Or was he a playwright? I tend to get them confused. -- Jack of Oz [pleasantries] 22:29, 21 July 2015 (UTC) [reply]
You're obviously confusing him with the impresario, Spectacles. {The poster formerly known as 87.81.230.195} 212.95.237.92 (talk) 12:22, 22 July 2015 (UTC)[reply]
There is a Biblical proscription against castration (Deuteronomy 23:1); "No one who has been emasculated by crushing or cutting may enter the assembly of the LORD" {New International Version), or the King James Bible which puts it rather more graphically; "He that is wounded in the stones, or hath his privy member cut off, shall not enter into the congregation of the LORD". However, Christians don't always treat the injunctions of Deuteronomy as binding. In the Byzantine Empire, parents would have their sons castrated in childhood to give them a chance of a lucrative career as an official in the Imperial court, and priesthood was open to eunuchs too (see The Eunuch in Byzantine History and Society). One example was Theophylact of Constantinople who "...was considered old enough to discharge his duties as patriarch (still he was still only sixteen years old). At this time or before he was castrated to help his career in the church." Not mentioned in our article, but Methodios I of Constantinople was also a eunuch; he displayed the evidence in court when appearing on a charge of seduction of a woman. In order to avoid the ban on castration by the Council of Nicea mentioned by User:Ian.thomson above, Methodios claimed that it had happened miraculously. [8] God moves in a mysterious way. This is perhaps the oddest subject I've ever researched for the Reference Desk; one never knows what will appear next. Alansplodge (talk) 18:47, 21 July 2015 (UTC)[reply]

Homosexual slavery in the United States

Resolved

How common was homosexual slavery in the American South before 1860? Are there any works of fiction (say, films) on the subject? --217.118.86.102 (talk) 15:55, 21 July 2015 (UTC)[reply]

Sources to help answer your historical question are cited in:
Foster, Thomas A. (September 2011). "The Sexual Abuse of Black Men under American Slavery". Journal of the History of Sexuality. 20 (3): 445–464.. Libraries and institutions offering access or simply Register & Read for free read-only access (no printing or saving) to three articles every two weeks (seventy-eight per year).
Foster also edited a collection with a few relevant articles and a much larger bibliography:
Foster, Thomas A. (2007). Histories of Same-Sex Sexuality in Early America. New York: New York Univ. Press. ISBN 978-0-8147-2750-8..
Paulscrawl (talk) 20:56, 21 July 2015 (UTC)[reply]
For works of fiction on the subject, including film adaptations, see:
Bibler, Michael P. (2009). Cotton's Queer Relations: Same-Sex Intimacy and the Literature of the Southern Plantation, 1936-1968. Charlottesville: University of Virginia Press. ISBN 978-0813927923.. Available in academic libraries via Project Muse: http://muse.jhu.edu/books/9780813929842?auth=0
The author's introduction states the case of current scholarship:
We simply do not know how many homoerotic or homosexual relationships might have flourished between men and women living and working on a plantation before or after the Civil War.1 Unfortunately, studies of same-sex relations in southern literature are similarly scarce, with only a few articles devoted to homoeroticism in works of plantation literature.2 (p 2)
Both sentences are footnoted, indicating references for both your historical and your literary question are readily available.
Paulscrawl (talk) 22:34, 21 July 2015 (UTC)[reply]
Thanks for your information. Good to know that the subject is no longer taboo in the US. --217.118.86.102 (talk) 07:46, 22 July 2015 (UTC)[reply]

Glad to know this information was helpful. Thanks to an editor on Wikipedia's Resource Exchange, I can now access the two footnotes cited following each of the two sentences just above, documenting the scholarly research on the historical and literary aspects respectively.

1. Most of the research into same-sex relationships in the South has paid attention to the twentieth century, as indicated in Sears, Growing Up Gay in the South and Lonely Hunters; Dews and Law, eds., Out in the South; Howard, Men Like That; and Howard, ed., Carryin’ On. Somerville’s Queering the Color Line discusses the connections between racial segregation and the ideological construction of homosexual bodies at the end of the nineteenth century but does not examine those connections in relation to the postbellum plantation. Carryin’ On contains a few articles about the nineteenth-century South, including Martin Duberman’s essay “‘Writhing Bedfellows’ in Antebellum South Carolina: Historical Interpretation and the Politics of Evidence” (153–68). Yet this inclusion supports my claim about the paucity of queer southern historical research, for Duberman’s essay was first published in Journal of Homosexuality in 1980–81, and reprinted again in Duberman, Vicinus, and Chauncey, eds., Hidden from History. While this essay is certainly important and insightful, the fact that it keeps getting republished in anthologies that intend to introduce something new to the study of sexuality exposes the absence of much else written about the South. In a provocative footnote to a recent essay, Noel Polk and Richard Godden briefly address the scant instances in historical records when white masters appeared to have sexually exploited their male slaves (“Reading the Ledgers,” 308 n. 7). But perhaps the best-known description of an antebellum sexual encounter between two people of the same sex appears at the end of Harriet Jacobs’s slave narrative Incidents in the Life of a Slave Girl, where she describes the humiliating sexual domination experienced by Luke at the hands of his corrupt master (192). For an interesting reading that situates this narrative with the history of homosexuality, see Abdur-Rahman, “The Strangest Freaks of Despotism.”

2. The only book-length study devoted to queer readings of southern texts is Richards’s Lovers and Beloveds, although Gwin’s The Woman in the Red Dress also discusses queerness in some southern works. Articles that consider plantation literature within a queer framework include Gebhard, “Reconstructing Southern Manhood”; and Forman, “This Promiscuous Housekeeping.”

Paulscrawl (talk) 14:49, 22 July 2015 (UTC)[reply]

Slavery and US currency

Which of the men currently shown on US currency owned slaves? Edison (talk) 15:56, 21 July 2015 (UTC)[reply]

(edit conflict)And except McKinley, but good luck getting one of those.
If we're counting coins, then we still have Jefferson on the nickel and Washington on the quarter for common use coins. Roosevelt and Lincoln are the two that didn't own slaves (so half and half for common use coins). Half and half there.
Of coins that don't see common use but are still circulated, we've got only non-owners: Kennedy, Eisenhower, Susan B. Anthony (who actually wrote antislavery petitions), and Sacagawea.
For the Presidential $1 Coin Program (can't say that it has caught on), most of the ones before Lincoln were slave owners except James Buchanan (who was still pro-slavery despite apparently not having any), Franklin Pierce (who was personally opposed to slavery but legally on the fence), Millard Fillmore (again personally anti-slave but wanted southern votes), John Quincy Adams, and John Adams. Of the presidents after Lincoln, none were slave owners except Andrew Johnson (who was generally a dick to former slaves afterward) and Ulysses S. Grant (technically his father-in-law's, and he didn't actively try to screw former slaves over, but still). Overall, 12 slave owners and 24 non-owners.
Overall, 14 slave-owners on circulating coins, 30 non-owners on circulating coins; though the non-owners are on less common coins. Ian.thomson (talk) 16:44, 21 July 2015 (UTC)[reply]
Cents and dimes are pretty common. Also, if Grant's father-in-law had slaves, that doesn't make Grant a slave owner. ←Baseball Bugs What's up, Doc? carrots16:47, 21 July 2015 (UTC)[reply]
Re cents and dimes: true, but that's 2/30 of the non-owners.
Re Grant: It's iffy, though. He attempted to profit directly from slavery, using his father-in-law's slaves. If we're going to put an asterisk next to Franklin because he freed his slaves and became an abolitionist, we need to put an asterisk next to Grant if we're going to count him as a non-owner because he probably would have been if it wasn't for outside factors. Ian.thomson (talk) 16:54, 21 July 2015 (UTC)[reply]
I stand corrected. Grant apparently acquired at least one slave. As regards cents and dimes, you said "the non-owners are on less common coins." Cents and dimes are very common coins. ←Baseball Bugs What's up, Doc? carrots17:01, 21 July 2015 (UTC)[reply]
If we're doing all circulating coins, there are also the state quarters to take into account. The slave owners on those are: Caesar Rodney (Delaware); Washington (also on other currency), James Monroe, Edward Hand and possibly others depicted in Washington Crossing the Delaware (New Jersey); and William Clark and Meriwether Lewis (Missouri; although I'm not sure whether Lewis ever owned slaves personally, he supervised his mother's plantation) - the non-slaveowners are the Wright Brothers (North Carolina); Neil Armstrong (Ohio); Lincoln (Illinois); Helen Keller (Alabama); John Muir (California); and Duke Ellington (DC). York, also depicted on the Missouri coin, was a slave. Kamehameha I, depicted on the Hawaiian coin, did not own slaves in the continental US sense, but the Kingdom of Hawaii had a strict feudal caste system with Kamehameha at the tip and a class of untouchables forced to work for chiefs - whether or not that counts as slaveowning for the purposes of the question is debatable. The South Dakota coin shows Mount Rushmore, which depicts Theodore Roosevelt (not a slaveowner) plus three presidents who appear on other currency. Smurrayinchester 09:29, 22 July 2015 (UTC)[reply]
(There are also some generic individuals on state quarters. The Massachusetts Minuteman could theoretically have been a slave owner (slavery was common in the state until the 1780s) - all the other images seem to postdate the abolition of slavery in their state.) Smurrayinchester 09:49, 22 July 2015 (UTC)[reply]
Looking over at the Alexander Hamilton article, I can't actually find anything saying he owned slaves. Some historians said he was an abolitionist, and he argued that the Brits were going to arm the slaves because the slaves are just as cunning as white people. Hamilton also suggested that they use this strategy against the Brits: buy their freedom and arm them.
That means 5 of 7 on banknotes were slave owners. Ian.thomson (talk) 23:57, 21 July 2015 (UTC)[reply]
I don't think Woodrow Wilson owned slaves. He's on the $100000 bill. --Trovatore (talk) 00:03, 22 July 2015 (UTC)[reply]
That's not in circulation, though. Non-circulating currency is a can of worms. Ian.thomson (talk) 00:11, 22 July 2015 (UTC)[reply]
(edit conflict)Recounted them off just figures (removing duplicates like Washington or Jefferson), 13 slave owners and 27 non-owners on circulating currency. With duplicates, 17 slave owners, 28 non-owners on circulating currency. Ian.thomson (talk) 00:10, 22 July 2015 (UTC)[reply]
If we're splitting hairs, Sacagawea and Susan B. Anthony weren't men. And Susan B. Anthony Man wasn't on any currency. InedibleHulk (talk) 05:35, July 22, 2015 (UTC)
Hamilton did not own slaves, technically, but his mother left him a slave when she died. A court ruled that he was not her legitimate offspring, since her divorce from her first husband was not recognized as legitimate, so boy Hamilton did not get his bequeathed slave. He married into money, and profitted from the labor of his wife's slaves. Edison (talk) 19:18, 24 July 2015 (UTC)[reply]

Shackles and leg irons.

How long can female prisoners in the US be legally restrained in leg irons and/or handcuffs? --Constiniolp (talk) 19:40, 21 July 2015 (UTC)[reply]

Presuming there are laws governing such things, it is likely the laws vary from state to state. ←Baseball Bugs What's up, Doc? carrots20:12, 21 July 2015 (UTC)[reply]

Germans

During ww2 Japanese Americans were interned but German Americans left alone even though German Americans were a larger community. Why did they detain the smaller community, but remained unconcerned about the larger community? 84.13.153.27 (talk) 20:02, 21 July 2015 (UTC)[reply]

As noted in Internment of German Americans, they were not "left alone", but they were interned out of proportion to the Japanese internment, as you suggest. ←Baseball Bugs What's up, Doc? carrots20:15, 21 July 2015 (UTC)[reply]
See also racism. --Jayron32 01:27, 22 July 2015 (UTC)[reply]
Do also note that the German American population was largely integrated in (White) American society. Today (according to Wikipedia...) 15.2% of U.S. population has German ancestry. That percentage was probably higher at the time of WWII. Apart from being unjust, systematic incarceration of German Americans would have been practically impossible to carry out. --Soman (talk) 10:47, 22 July 2015 (UTC)[reply]
That there were less people with Japanese ancestors who were inside the U.S. than those with German ancestors doesn't make the internment of those people any more tolerable... --Jayron32 21:27, 22 July 2015 (UTC)[reply]
As a good illustration of this, Dwight Eisenhower was ethnically German. "Eisenhower" is an Anglicized form of the German name "Eisenhauer". --108.38.204.15 (talk) 12:34, 22 July 2015 (UTC)[reply]
Also, it is worth noting that some of the assimilation was a result of earlier anti-immigrant sentiment. There was a notable amount of anti-German sentiment in the U.S. during World War I, even before the U.S. entered the war. People speaking Germanic languages (who were often more recent immigrants) were pressured to stop, German-derived names of places and things were changed, etc. --108.38.204.15 (talk) 12:45, 22 July 2015 (UTC)[reply]
Not to split hairs, but English is a Germanic language, as are Dutch, Swedish, and Norwegian. --Stephan Schulz (talk) 14:46, 22 July 2015 (UTC)[reply]
Without splitting Herrs again - also Icelandic, Danish and others (noting "Norwegian" is more than one language). Collect (talk) 15:26, 22 July 2015 (UTC)[reply]
Or, as I've heard, there is only one Norse language, but the Danes don't know how to pronounce it, and the Swedes don't know how to spell it ;-). --Stephan Schulz (talk) 15:33, 22 July 2015 (UTC)[reply]
majority ethnicity by county, 2000
  • Just anecdotal, but according to my father there was blackballing of people with germanic sounding accents or names in the Philadelphia shipyards during the war, and his grandmother's restaurant went out of business due he believes in part to the mistaken perception that her last name was German. But as has been noted above, the largest proportion of American ethnicity has long been of German descent. μηδείς (talk) 19:30, 22 July 2015 (UTC)[reply]
    • I think you are looking for the adjective "German", not "Germanic". See above. English sounds Germanic. --Stephan Schulz (talk)
No, I said exactly what I meant, everyone in my family except my mother studied German in school, my father knows the difference between Danes, Dutch, and Germans (although a lot of Americans don't, nor the difference between Swiss and Swedes), and he meant exactly what I reported; there was some prejudice against the non-Deutsch when their names or accents were Germanic. μηδείς (talk) 02:19, 23 July 2015 (UTC) [reply]
Well, in this case you have me confused. English is a Germanic language, so English sounds Germanic. Do you mean to say that there would be prejudice against, say, Mr. Miller? Or do you want to restrict this to Non-English Germanic languages? Or only to languages which sound more-or-less like German or share e.g. Umlauts with it? --Stephan Schulz (talk) 07:15, 23 July 2015 (UTC) [reply]
Stephan, you seem to be expecting me to go to the length of saying compared to "non-English Germanic accents" when that is already implied in the question. I can see that this might be a justified pet peeve, but you really don't need to project it on the English ref desk. Also, to say that English, which is quite a phonetic outlier among the Germanic languages, with its diphthongs, th's, r's, w's, loss of desinences, and so forth, "sounds Germanic" is about as true as saying that French "sounds" like a Latin (i.e., Romance) language. It's true by definition, perhaps, but not very informative (Old Prussian is a lot more "Germanic" sounding than English to my ears), and not a point it really seemed necessary to make. μηδείς (talk) 21:36, 23 July 2015 (UTC)[reply]
The reason was more pragmatics than racism. Japanese were despised back then, and so were Germans. There were treated differently for a variety of reasons. At Internment of Japanese Americans you see the full rationale for the decision, which has not much to do with the previous contributions above mine. --Scicurious (talk) 05:21, 23 July 2015 (UTC)[reply]
I imagine two factors were that the Germans hadn't attacked the United States directly at that time, nor were Americans afraid of invasion by them. The bombardment of Ellwood (by a Japanese submarine) article claims "Though damage was minimal, the event was key in triggering the West Coast invasion scare and influenced the decision to intern Japanese-Americans." Clarityfiend (talk) 08:24, 23 July 2015 (UTC)[reply]
During WW2 the US did not have a policy of interning German-American US citizens,, but they interned German nationals. In some reported cases, US citizens voluntarily joined their non-citizen family members in the internment. It would have made no sense to do otherwise than round up loyal subjects of an enemy power; how could a country at war allow citizens of the warring country to travel around and go where they wished to do what they saw fit, possibly including sabotage and espionage? Of course some enemy civilians would have been given special status as refugees, and some famous artists or religious people might have been given special privileges, like older and more genteel times when "paroles" were granted to captured enemy soldiers who promised not to fight any more. The US citizens of German descent were not thought of as disloyal, although accounts of actual spy rings show that it was often possible to recruit them as spies, on the basis of "loyalty to the fatherland" or through threats of harm to their family in the "old country." In fact there were thousands of Bund members who marched around in militaristic uniforms displaying their devotion to Germany and Hitler up until the outbreak of war, at which time they gave up outward displays of disloyalty to America. after the outbreak of war there was for some reason greater suspicion of those of Japanese ancestry having loyalty to the home country and the Emperor. The American Bund Germans marched around in the 1930's until America's entry in the war, but did Japanese-American US citizens similarly march round declaring their support of Tojo and Japanese militarism, or their loyalty to the Emperor? Did they display divided loyalty by posting signs in their communities saying "We Stand With Japan?" Edison (talk) 18:59, 24 July 2015 (UTC)[reply]
Can I point out that support for Hitler, Nazism, or the Reich before the US declaration of war was not necessarily disloyalty? μηδείς (talk) 20:31, 25 July 2015 (UTC)[reply]

July 22

bronze age India

what type of bed and mattress were used in bronze age India (e.g. the Buddhas time)

Thank you!!

Those aren't the same periods. The Buddha lived from c. 563 BCE or c. 480 BCE to c. 483 BCE or c. 400 BCE, according to our article. The Indian Bronze Age ran from about 3000BC to 1000BC. (Unfortunately, I don't know the answer for either period). 62.172.108.24 (talk) 13:13, 22 July 2015 (UTC)[reply]
Found this quite difficult to research, but try Manjaa. (No refs or sources.) Perhaps one of the other language wikipedias has more. 196.213.35.146 (talk) 13:23, 22 July 2015 (UTC)[reply]
There is quite a detailed description in the Vinaya texts here. I'm not quite certain when these were written (maybe a few hundred years after the Buddha's time?) but they purport to be contemporary descriptions for his time. Anyway, if this is the time period you mean:
The monks initially slept on bare dirt floors, but eventually moved on to grass mats and eventually beds, described as bedsteads made of laths of split bamboo which were raised at least eight inches off the ground as a deterrent to snakebite. The bedframe was criss-crossed with a lattice of string. The mattress is described as stuffed with cotton but pillows were made of skin and stuffed with wool, or cotton cloth, or bark, or grass, or leaves. Sometimes the coverings were cloth, and were coloured with patches of dye. Click through for many more details. 184.147.131.217 (talk) 13:31, 22 July 2015 (UTC)[reply]
For the Bronze Age period, toy beds have been found at Kalibangan. According to this book and this book they had corner posts and the authors speculate the real beds were made of wood with a rope lattice, just like 196.213.35.146's link above. This describes a toy bed with uprights at head and foot. 184.147.131.217 (talk) 13:59, 22 July 2015 (UTC)[reply]
This site claims they are an ancient furniture style. "Ancient" is a bit vague but there is a ref. See http://www.stringbedco.com/history.html 196.213.35.146 (talk) 13:35, 22 July 2015 (UTC)[reply]

States vs Non-state societies vs something inbetween

I've been looking at our articles on States and Stateless societys. While both articles seem to imply that all societies are one or the other, the actual definitions used seem to leave a large gap for various historic societies that fit neither definition:

  • Stateless society mostly talks about hunter-gatherer societies with little division of labour, plus a few ancient urbanised civilizations (e.g. Harappan civilization for which no evidence of governments have been found.
  • State (polity) days there is no universally agreed definition of the concept of a state, but says that the most commonly accepted is Max Weber's "a compulsory political organization with a centralized government that maintains a monopoly of the legitimate use of force within a certain territory". (Monopoly on violence explains that a) this means the state is the sole source of the right to the use of force, not necessarily the only entity allowed to use it, and b) doesn't apply to all historic states).

This seems to me to leave a bit of a gap in definitions: there are a lot of pre-modern societies that had more division of labour, government, etc than the sort of societies described in Stateless society, but which don't meet Weber's definition of a (contempory) state. Of course, as pointed out previously, Weber says his definition didn't apply to past states, but that makes it pretty useless for distinguishing between stateless and "old" state societies. Given that, a) what level of organisation, government, taxation/tribute collection, etc are mark the transition between non-state and state socity, and b) do the respective articles need improving to exlain this? (I'm thinking particularly of the "Types of state" section, which currently only talks about sovereign vs. (con)federated states, but should probably talk also talk about various historic state types, eg. feudal, imperial, city states, etc). Iapetus (talk) 15:22, 22 July 2015 (UTC)[reply]

Agreed, articles could use development. Probably best outlined on articles' Talk page first. Weber certainly does not have the last word on political anthropology. I've added two references to that article for sources on current scholarly thinking you'll likely find at a nearby academic library:
  • Abélès, Marc (2010) "State" in Alan Barnard and Jonathan Spencer (eds.), The Routledge Encyclopedia of Social and Cultural Anthropology, 2nd. ed., London and New York: Routledge, pp. 666-670. ISBN 978-0-415-40978-0 Includes bibliography
  • Sharma, Aradhana and Akhil Gupta (eds.) (2006) The Anthropology of the State: A Reader, Malden, MA; Oxford: Blackwell. ISBN 978-1-4051-1468-4 Includes Weber and rejoinders. Good intro.

-- Paulscrawl (talk) 17:06, 22 July 2015 (UTC)[reply]

  • The OP presents what is commonly called the fallacy of the excluded middle; They ask about a binary, single-axis classification scheme which in reality is neither single-axis, nor binary. The organization of society shouldn't be thought of as either "state" or "non-state", but as series of multi-dimensional continuums that define such matters as who makes decisions for the society, how those decisions are made, how those decisions are enforced, etc. etc. There's way too much to say there are only two options, and either a group of people is a "state society" or a "non-state society". I know the OP defines their question with "something in between", but still looks for binary thinking, i.e. looking for a bold line that defines the "transition between state and non-state". I'm not sure that's a productive way of thinking. Instead, one might want to look at a continuum between "absolte anarchy" and "absolute control" and have those extremes be asymptotes that define the edges of the continuum: defined edges which are themselves unattainable, and then look to figure out where a society fits on that continuum (also keeping in mind there are dozens of other intersecting continuums to consider) rather than saying "state" or "stateless". As soon as a group of people has binding decisions made for their future action, they've entered the continuum. --Jayron32 21:24, 22 July 2015 (UTC)[reply]
Funny, I call that a false dichotomy. Then there's the Law of excluded middle, one of the Three classic laws of thought. I understand this is just terminology, but thought it was funny that the same concept (and name!) is considered a law in some circumstances and a fallacy in others. SemanticMantis (talk) 23:07, 22 July 2015 (UTC)[reply]
Point taken. (When I started writing that question, I don't think I was thinking so binarily, and in fact was probably going to criticise the articles for being too binary. But I took so long writing it, due to doing other things at the same time that I rather forgot where I was going with it and what my original question was going to be). Still, the false dichotomy / fallacy of the excluded middle problem seems to be quite ingrained in the articles, with one seeming to treat "stateless society" as a very narrow thing, with everything else being a state, while the other treats "state" as a very narrow thing and (at least by implication) everything else being a stateless society. Which together leaves a broad continuum of things that they don't agree what to call. Iapetus (talk) 09:39, 23 July 2015 (UTC)[reply]

Cutting the thumbs of Bengali weavers: reliable source?

Indian politician Shashi Tharoor claims in this "viewpoint" piece that "the British" cut off the thumbs of Bengali weavers because they were competing with British industrial textiles: Britain's response was to cut off the thumbs of Bengali weavers, break their looms and impose duties and tariffs on Indian cloth, while flooding India and the world with cheaper fabric from the new satanic steam mills of Britain. You can see that same fellow make the same claim at the Oxford Union here or here (that's 1 minute 58 seconds into the video if no time offset format works for you) only here he claims that "Britain" "smashed" their thumbs and did not cut them. On the other hand WP article Mahua Dabar states: By local legend the town was partly settled by Bengali textile workers fleeing British persecution in the 1830s. According to this legend, the East India Company had mutilated the skilled workers by chopping off their thumbs, making them unable to work. However, there is no historical evidence for this event, and most academic historians believe it to be a myth that arose, either by adaptation from the story of Ekalavya in the Mahābhārata, or from a mistaken quotation from a contemporary British source reporting possible self-mutilation by Bengali workers to break their indenture. The WP article is only about one town. How about Bengal in general? This source, this source and this source seem to imply this is just a myth. On the other hand this source, this source and this source mention otherwise unspecified "archives at the National Library in Kolkata" that confirm the story is true. On the face of it, the story seems nonsensical especially the way that Indian politician presents it. After all if you can produce cheaper textiles you do not need to maim anyone. The traditional weavers will just go out of business. But then we know how evil and cruel "the British" were, so maybe, who knows, they just liked to cut off thumbs for the hell of it. What is the deal? Are there any reliable sources for that story or not? How about those "archives at the National Library in Kolkata"? Contact Basemetal here 17:58, 22 July 2015 (UTC)[reply]

Gurcharan Singh Bhikhi (Sidki)

Hi there, I'm dealing with a problem article at Gurcharan Singh Bhikhi (Sidki). The article was created by a grandchild of the subject, a Punjabi poet, so there is a bit of a COI issue here. The entirety of the article is unsourced and I'm having trouble finding references that establish the subject's notability and previous attempts to ask WikiProject Sikhism (which I believe is a dead Wikiproject) have not been successful. If anyone can dig up anything of note, that would be sweet. Thank you. Cyphoidbomb (talk) 20:45, 22 July 2015 (UTC)[reply]

Japanese consumers

Why do consumers in Japan seem to prefer physical media whether it's books, CDs, blu Rays etc over electronic media such as e books, VOD/music streaming etc? There also seem to be other phenomena which is different to western consumer culture such as the widespread use of fax to this day and also the lack of use of social media by the average consumer. 90.201.189.237 (talk) 20:57, 22 July 2015 (UTC)[reply]

See Complex question for why we cannot answer you question in any meaningful way. You've asked "why does X happen", but you have first not established that X is true. In other words, everything you say after "why do..." is itself not yet shown to be true, so we cannot answer the why do question until you've first presented evidence that the information that comes after it is true first. --Jayron32 21:15, 22 July 2015 (UTC)[reply]
How do I present evidence? Are trustworthy news sources etc ok? 90.201.189.237 (talk) 21:29, 22 July 2015 (UTC)[reply]
Yes of course. Contact Basemetal here 21:49, 22 July 2015 (UTC)[reply]
(edit conflict) Here are a number of sources relating to your question: BBC, NY Times, The Guardian, Reuters. Those, some blogs I've read by ESL teachers (which, granted, would fail WP:RS), as well as preparation materials from ESL-teaching companies I applied to present a pretty solid image that while Japan advances new technology, it's not out of neophilia. If a fax machine made in the Thatcher administration still works, and the people you do business with are likewise using fax machines, there's no reason to throw it out or replace it with something that your business partners might not be using.
Purely anecdotal, but some friends of mine have resistance to digital releases for Japanese games because the manufacturers are convinced that it'll only lead to pirating.
As for social media, it is present, but focuses more on anonymity. I'd have to dig around for the blogs that explain that (been a few years), but the reasoning I can recall is that most Japanese people know they're not celebrities (...unlike one of my cousin's kids), and anyone who would care to keep up with them like that will either know their online pseudonym or actually see them in person. Ian.thomson (talk) 21:40, 22 July 2015 (UTC)[reply]
Yes of course. Contact Basemetal here 21:49, 22 July 2015 (UTC)[reply]
In principle an answer here is not supposed to be based on our personal speculations but on reliable sources. What difference does it make if the OP posts a source for his statement? Either there is a reliable source to answer his question or there isn't. Or maybe you mean "before I start wasting my time to try and locate sources to answer your question, maybe you should first present evidence that my efforts have any chance of being worthwhile"? Contact Basemetal here 21:49, 22 July 2015 (UTC)[reply]
To help the OP's question in a more general sense I will answer "why do some people prefer paper books instead of Digital books?". When reading a paper book there are experiences that you would not obtain from an E-book. Scanning from one page to another, holding the book in your hands, turning each page all provide stimuli to the brain that affects your imagination and overall experience. The culmination of all these micro variables give rise to a macro experience. Some quotes from a Japanese man: E-books lack character, books are not something that you just read words in, They're also a tool to adjust your senses.

When I don’t feel well, I’ll stare at a page for ever before realizing I haven’t absorbed a word. When that happens, I try to understand why. What’s gotten in my way? On the other hand there are books I can take in effortlessly, no matter how awful I’m feeling. Why do those books draw me in? I think it may be a sort of mental tuning. It’s the feeling of the paper against my fingers, that familiar smell of pulp and glue, a momentary stimulation to my brain when I turn each page. These sensations regulate and focus my brain, they make it work better.” Void burn (talk) 01:19, 23 July 2015 (UTC)[reply]

The King's step-children

Fast forward to the demise of Queen Elizabeth II and the accession of her son Charles (whatever regnal name he may take).

He has 2 step-children. When was the last time a British monarch had a step-child? I suspect we'd need to go back more than 250 years. -- Jack of Oz [pleasantries] 22:43, 22 July 2015 (UTC)[reply]

The last British monarch to have a step child I can find was Henry VIII of England, who had a posthumous step child in the person of Mary Seymour, who was the daughter of Catherine Parr from her second marriage. If that counts; I'm not sure that children born after the end of a marriage to one's spouse count as step-children though. Still looking further back. --Jayron32 22:56, 22 July 2015 (UTC)[reply]
If you don't want to count that as "step children", the last British monarch to have actual step children (meaning children of their spouse from a prior marriage) was Edward IV of England, who's wife Elizabeth Woodville had two children from her prior marriage to John Grey of Groby; those being Thomas Grey, 1st Marquess of Dorset (a direct ancestor of Lady Jane Grey) and Richard Grey. --Jayron32 23:02, 22 July 2015 (UTC)[reply]
No, I've never heard of the children of a person's relict or ex-spouse, who were born to their new spouse after the initial person's death or the end of the marriage, referred to as step-children. If you're right about Edward IV, and I suspect you are, that means my initial guesstimate of over 250 years ago was over 50% too short. Thanks. -- Jack of Oz [pleasantries] 01:09, 23 July 2015 (UTC)[reply]
I think that Philip_II_of_Spain#Family says that Mary I had a stepson. Her husband Philip had a nine-year-old son from his first marriage when he married Mary. 184.147.131.217 (talk) 02:07, 23 July 2015 (UTC)[reply]
Ah yes, Carlos, Prince of Asturias became the stepson of Mary I of England upon her marriage to Philip II in 1556. That's still 450 years ago. This would be a great pub quiz question: Which opera by Verdi was about the stepson of Bloody Mary? -- Jack of Oz [pleasantries] 02:26, 23 July 2015 (UTC)[reply]

July 23

Conversion to Christianity in the 1700s - Robinson Crusoe

I am currently reading a novel, Robinson Crusoe, leisurely. I noticed that Xury could be freed in ten years, if he turned Christian. Were there any barriers to religious conversion back then to keep people in servitude? How could one prove oneself to be a convert? Were there consequences for lying? If one did convert, who would be able to solemnize the baptism? 71.79.234.132 (talk) 02:39, 23 July 2015 (UTC)[reply]

Even in the 1700s, it would depend on the strain of Christianity. Everyone from Anglicans to Roman Catholics to Anabaptists would have had different requirements for conversion to Christianity. --Jayron32 02:45, 23 July 2015 (UTC)[reply]

Fine. Narrow it to Daniel Defoe's Presbyterianism then. 71.79.234.132 (talk) 02:47, 23 July 2015 (UTC)[reply]

Presbyterianism#Sacraments indicates that baptism by sprinkling or pouring is all that is required. One cannot prove earnestness, however, as the concept of a mind-reading device had not been invented in the 1700s. I'm not sure one exists today even... --Jayron32 02:54, 23 July 2015 (UTC)[reply]
So, basically, Xury will just have to get people to trust him. 71.79.234.132 (talk) 02:58, 23 July 2015 (UTC)[reply]
Depends who the people. Testimony of faith comes with religious observance: convert. --Askedonty (talk) 07:50, 23 July 2015 (UTC)[reply]

Read all about it: a reliable source for the theological background assumed by Daniel Defoe for the several religious conversions recounted in Robinson Crusoe is:

Greif, Martin J. (Summer 1966). "The Conversion of Robinson Crusoe". Studies in English Literature, 1500-1900. 6 (3): 551–574.
"Libraries and institutions offering access".. JSTOR's "Register & Read". offers free read-only access.

Paulscrawl (talk) 05:53, 23 July 2015 (UTC)[reply]

Are price tags in a store legally binding?

So, say I walk into a supermarket (in the USA), and an item (let's just say a can of corn) is marked with a price of 2 cents. When I go to the cash register, the clerk says: "that can costs 99 cents". Is there any legally binding reason to sell me the can for 2 cents? Does that sign on the item (the price mark) constitute any type of oral/written contract? 2602:252:D13:6D70:21BC:1B64:824C:5D8F (talk) 03:00, 23 July 2015 (UTC)[reply]

Not in Canada, according to this [9]. I suspect this powerpoint [10] file says the same thing for the USA. Also not a contract in the UK according to this [11]. Also not a contract in NZ according to here [12]. The key legal concept seems to be that price tag is considered an Invitation to treat, not a binding legal contract. Contract#Invitation_to_treat has a nice short summary. (Don't bother looking at Price_Tag that's entirely unrelated, and I don't see a decent disambiguation page for the concept.) SemanticMantis (talk) 03:58, 23 July 2015 (UTC)[reply]
It depends on your jurisdiction. What your hypothetical clerk did is explicitly against the law in California per section 12024.2 of the California Business and Professions Code. I have no idea what the law is in other states, or whether investigators actually look into these sorts of violations, or what case law has to say about the limitations of this law's application. Someguy1221 (talk) 04:07, 23 July 2015 (UTC)[reply]
(ec)This will depend on the state, or even the municipality. A sale price is typically honored in NY & NJ if it is displayed on the shelf but not rung at the counter. (I.e, this has happened to me dozens of times, and the clerk has always honored the price at the shelf if lower than the price at the register.) The NYC consumer protection guide says "Any PRICE QUOTED in an ad must match the actual purchase price" but it also says "Watch out for FOOTNOTES AND ASTERISKS (“*”). The “fine print” in an advertisement sometimes changes an offer made in the large print." You do see adds with the very fineprint boilerplate that misprinted prices will not be honored, which has happened with lottery tickets, (a recent example from New Mexico). For how NYC law addresses the seeming above contradiction in the guide (which is not a copy of the very confusing law) I will refer you to a lawyer. μηδείς (talk) 04:20, 23 July 2015 (UTC)[reply]
A sale price is typically honored in NY & NJ if it is displayed on the shelf but not rung at the counter. Yes, it is often "honored". But are they (the store owners) doing that out of goodwill or legal obligation? 2602:252:D13:6D70:21BC:1B64:824C:5D8F (talk) 05:21, 23 July 2015 (UTC)[reply]
The New York law is far less clear than the California law, and is not at neatly contained in one section. It sounds like there are civil penalties imposed by state officials if more than 2% of items in the store are incorrectly priced, as far as I can read from the New York Agriculture and Markets law section 197b. Someguy1221 (talk) 06:05, 23 July 2015 (UTC)[reply]
Yes, unfortunately I did take some time looking into this, but there was nothing outright I could quote that said plainly that they must honor the advertised price. Sorry. μηδείς (talk) 00:32, 24 July 2015 (UTC)[reply]

Specifically with collectibles, the fact a comic book has "10 cents" in the corner (for example) does not obligate a seller to honor that price <g>. Collect (talk) 16:54, 23 July 2015 (UTC)[reply]

You are mixing apples and oranges. If a comic book has the notation "ten cents" on the cover (placed there by the publishers), that is not the "price tag" that has been placed there by the seller of the item. Two very different things. If a seller places a price tag, it is (presumably) the price that they are charging for the item. Joseph A. Spadaro (talk) 18:54, 23 July 2015 (UTC)[reply]

Capital of Denmark

Which cities were the capitals of Denmark before Copenhagen? --Ghirla-трёп- 11:38, 23 July 2015 (UTC)[reply]

Roskilde. I think that's pretty much it. Before Roskilde was capital (C11th) there wasn't really a Danish state. Ever since the capital moved to Copenhagen in 1443 (but note our Copenhagen article says 'by about 1416') Copenhagen has been the capital, whether of the Kalmar Union, of Denmark-Norway or of modern Denmark. If you're making a list, though, be sure to add Gainsborough, Lincolnshire to it! - http://www.bbc.com/news/uk-england-lincolnshire-21386473 - Cucumber Mike (talk) 14:11, 23 July 2015 (UTC)[reply]
Actually, I wouldn't necessarily add Gainsborough. It is true that it served as tcenter of the administration of Danish-controlled England for a time, and that Sweyn Forkbeard used it as his base of operations, but I don't know that much formal administration of the Danish state occurred there. The North Sea Empire of Sweyn and Cnut really should be thought of as multiple states under the same ruler, each with it's own institutions. Both practically, and officially, England's capital was still Winchester, and Denmark's was Roskilde (after all, Cnut returned to Roskilde to get the crown of Denmark) and laws of Denmark were not in force in England and visa-versa. Likewise, when he took over Norway, Cnut was crowned at Nidaros and Norway, and Norway retained it's institutions as a separate country from there. --Jayron32 16:35, 23 July 2015 (UTC)[reply]
Another to add to the list Jelling was the capital of Gorm the Old, the first historically verifiable King of Denmark, as well as his son Harald Bluetooth. Harald moved the capital to the newly created Roskilde in the 980s, and then it was moved to Copenhagen in 1416, as the seat of Eric of Pomerania, first king of the Kalmar Union. Thus, the Danish capitals can be considered to be:
Prior to Gorm the Old, there was no historically verifiable Danish state... --Jayron32 16:45, 23 July 2015 (UTC)[reply]
The List of former national capitals contains only Roskilde. --Ghirla-трёп- 07:04, 24 July 2015 (UTC)[reply]
Well, part of the issue is defining when "Denmark" started, what a capital is, etc. etc. There are not universally agreed upon definitions for those ideas. --Jayron32 16:00, 24 July 2015 (UTC)[reply]

Also consider Fredericia ca. 1650 [13] originally intended as a capital city in Jutland by Christian IV due to its strategic location. Unfortunately, the Swedes also considered it to hold a strategic position. Collect (talk) 16:51, 23 July 2015 (UTC)[reply]

Stylised Dublin map/diagram

I want to represent some areas of County Dublin on a map or diagram. The areas are mainly, but not exclusively, within the M50, and cover all or parts of Dublin City Council, South Dublin and Dún Laoghaire–Rathdown. I want to be able to draw areas onto the map to show that, for instance, Tallaght is in one area and Clondalkin in another

I would like the map/diagram to:

  • Cover the above areas (outlying areas may be covered as well if necessary).
  • Show the approximate course of the River Liffey and M50. Phoenix Park is an optional extra.
  • Not have too much (any?) extra information/labels.
  • Be accurate enough that major areas of the city can be pinpointed (e.g. Tallaght, Finglas, Ballyfermot etc), but not so accurate that viewers can get hung up on exact geography - e.g. is the boundary between areas on this side of the road or the other?
  • Be a reasonable quality for viewing at 1024x768 and above.
  • Be available on a free-ish license including for modification and commercial purposes.

I'm ok with Photoshop and Illustrator, so if I had a basic blank map I can fill in the gaps.

Something like this would be perfect, but I don't want the junction blobs and road names, and it doesn't quite cover all the way down to Dun Laoghaire.

I've looked around on the internet but drawn a blank so far. Can anyone help find what I'm looking for? Many thanks - Cucumber Mike (talk) 13:59, 23 July 2015 (UTC)[reply]

You can extract this kind of thing from OpenStreetMap. There's a basic map here which you can copy under the Creative Commons licence, but if you want to make it more advanced (eg. to chose which roads to show) you'd need to use a tool like QGIS, which has a very steep learning curve. Smurrayinchester 14:11, 23 July 2015 (UTC)[reply]
(However, if you click on the "Layers" symbol there are a few pre-set map types you can look at. "Transport" or "MapQuest Open" might be good.) Smurrayinchester 14:15, 23 July 2015 (UTC)[reply]
That might actually work. I see there's an option to export as SVG. I'm not able to test it at the moment - do you know if I would be able to select roads and other features as elements (I think that's what they're called - shapes/paths) within an image editor? - Cucumber Mike (talk) 14:19, 23 July 2015 (UTC)[reply]
I think so, yes. Alternatively, if you want I can try extracting it in QGIS to separate the roads, river and boundary data when I'm back at my desktop. Smurrayinchester 14:44, 23 July 2015 (UTC)[reply]
Ok, perfect. In that case, I'll have a bash at exporting it and fiddling around at home later and let you know if I get stuck, if that's ok? I've got a few weeks to get it together, so no rush. - Cucumber Mike (talk) 15:00, 23 July 2015 (UTC)[reply]

Pearl Harbor

Is it true that some Americans believe the 2011 Tōhoku earthquake and tsunami was karma for Pearl Harbor? 62.37.237.16 (talk) 16:26, 23 July 2015 (UTC)[reply]

That question assumes that Americans believe in karma. Or that most Americans either do not know about the atomic bombings of Hiroshima and Nagasaki, or somehow think that those weren't enough -- Overall the question is inherently flawed.
Of course, if you want to ask "Is it true that some Americans believe (any random idea)," it is possible to find at least one or two who might think that idea is plausible. Are those few Americans representative or even noteworthy? No. Ian.thomson (talk) 16:31, 23 July 2015 (UTC)[reply]
It seems to be more than "one or two"; see Earthquake Was “Payback” for Pearl Harbor, Japanese Earthquake, Tsunami reveal American Stupidity, Insensitivity and Facebook Meme: Pearl Harbor Payback is a Bitch!. There's pages of stuff on Google. I'm not sure if it proves that the US has more morons on social media than anybody else, but certainly "some Americans believe". Alansplodge (talk) 16:59, 23 July 2015 (UTC)[reply]
Probably not as many morons as the ones who think we didn't land on the Moon.[14]Baseball Bugs What's up, Doc? carrots17:11, 23 July 2015 (UTC)[reply]
The sites given would fail WP:RS, the first two as blogs, the last as a forum. The first two are engaging the sort of sensationalism that lead a number of news sources to claim that Bagel head was popular in Japan. The last is questioning whether it really is a significant number or just a few random idiots. Ian.thomson (talk) 17:28, 23 July 2015 (UTC)[reply]
In one of the sites above some of the opinions seemed to lead to questioning the influence of Hollywood on the fragile minds of some of the related people. I googled "dire effects of Hollywood" and I found this: The-Effect-of-Hollywood-on-Historical-Perspective. However regarding hate culture itself I see its roots rather in a ( frustrated ) cult of the automobile than in that of the cinema. --Askedonty (talk) 18:43, 23 July 2015 (UTC)[reply]

Police asking "You OK? You seem very irritated" when giving out a ticket: purpose?

What could be the purpose of police in the US asking you (or telling you) "you seem very irritated" when giving you a ticket? I assume there is a pattern to the questions police are trained to ask. I mean how they go about giving you that ticket and what they tell you is scripted in advance and part of their training, isn't it? I could see how, for example, some questions could serve the purpose of trying to find out if there's something else going on (e.g. trying to see if there's drugs in the car, if a crime has taken place or is about to take place, etc), for example by getting you to unwittingly spill the beans or incriminate yourself to give them an excuse to dig a bit deeper. But in this particular case, as far as I can tell, there is only one thing that question can achieve, and that is to irritate that person even more. Any clues? If you know anything about how police are trained to act when giving out a ticket maybe you can help with this one. Contact Basemetal here 18:26, 23 July 2015 (UTC)[reply]

Depending on the size of the town/department, there may be little to no training. Even the most populous county in a state can have inadequate or improper training. Ian.thomson (talk) 18:37, 23 July 2015 (UTC)[reply]
Are police departments funded exclusively by local taxes? Contact Basemetal here 18:40, 23 July 2015 (UTC)[reply]
PS: I meant to include state police in my question. Contact Basemetal here 18:42, 23 July 2015 (UTC)[reply]
Police departments, like schools, are mostly locally funded by the county or state they answer to, with some help from the federal government (like the army handing some small-town departments leftover tanks, machine guns, and grenade launchers; or Obama funding body cameras)
(edit conflict)Even the sixth most populous county in the nation has inadequate training, and many departments that have "adequate" training focus more on dealing with violence encouraged by said training, instead of helping people (perhaps suffering from mental ailments or handicaps) going through terrible crises. Ian.thomson (talk) 18:48, 23 July 2015 (UTC)[reply]
Note: I made my posts as someone who is largely ignored by the police, even when going 20 mph over the speed limit with busted tail lights because I needed Little Debbie cakes and orange soda at 3 am. Ian.thomson (talk) 19:08, 23 July 2015 (UTC)[reply]
OTH, if there isn't enough money to properly train police, has anyone seen any proposals to spend some federal, state or local money on educating citizens and disseminating information (especially among the sections of the population that seem to be more "noticed" by police, e.g. those "driving while Black") on how to deal with police and especially poorly trained police? Maybe along the lines of this page or this page. But I mean a large scale effort funded with real money by the authorities. Can something like that ever be politically acceptable? On the one hand it would look like the authorities are warning citizens against their own police, i.e. their own employees. On the other there's been so many people hurt or losing their lives in encounters with law enforcement that it looks this would be a much better way to spend money. Contact Basemetal here 19:43, 23 July 2015 (UTC)[reply]
Educating the populace on how to avoid police brutality and racism instead of educating the police to not engage in said brutality and racism would probably come across as victim blaming and skewed priorities to most Americans (by those who acknowledge the problems, at any rate... The NRA blames shootings on everything but guns and acts as though any shooting that makes the news would have occurred without guns, and yet they still have members; so victim blaming and skewed priorities appears to be normal thinking for an unfortunate number of Americans.) Ian.thomson (talk) 19:58, 23 July 2015 (UTC)[reply]
Then as long as no one has the common sense of trying something else, people will continue to die in their encounters with police. Sometimes pragmatic solutions look bad but do the job better than "ideal" ones. Warning citizens is such another pragmatic approach. It is just saying "be very very careful when you deal with those guys". In a small way this is already being done, as you can see on the net. But obviously it does not reach everyone. I thought maybe a massive educational campaign, in schools, on mass media, social media, the net, etc would be money well spent. Unfortunately in politics symbolic trumps pragmatic in very many cases, even though to keep doing the same thing and expect different results is the essence of insanity, as people say. Look at the war on drugs. Well, at least some have proposed other approaches to the problem of illegal drugs. But, from what you're saying, I conclude no one has ever even suggested an approach to police brutality like the one I asked about here. Contact Basemetal here 15:56, 24 July 2015 (UTC)[reply]
But in this particular case, as far as I can tell, there is only one thing that question can achieve, and that is to irritate that person even more. I'm not sure I agree with that, Basemetal. It may cause the person to reveal why they're irritated, but I can't see how having that feedback would be irritating in itself, thus exacerbating their state. Giving feedback about a person's apparent emotional state is a common technique among counsellors etc, and it's designed to show empathy and create a bit of rapport, a window of trust where the other has the opportunity to open up. Now, if the apprehendee already has a distrust of police, that would admittedly complicate the picture. -- Jack of Oz [pleasantries] 20:10, 23 July 2015 (UTC)[reply]
"Upset" would be a better term. ←Baseball Bugs What's up, Doc? carrots20:32, 23 July 2015 (UTC)[reply]
"Well, you see officer... here I was, just driving along, minding my own business... when suddenly this policeman pulls me over... and now i am about to be given a damned ticket... So yeah... I guess you could say I'm a bit upset.
Imagine how the Police would react if civilians were to turn the question back on them... "I'm fine, officer... Are you OK?... you too seem a little irritated. I sure hope it's not something I said or did." Blueboar (talk) 21:25, 23 July 2015 (UTC)[reply]
Well Jack my thinking may have been influenced by one particular video, that of the arrest of Sandra Bland. I tried to figure out what on earth the state trooper was trying to achieve with that question. That prompted my asking this general question here, just in case this was some kind of general question they were trained to ask for some reason. But you may be right that, in general, such a question may produce different results, depending on circumstances, the way it is asked, etc. However I'm fairly sure that in this particular instance you will agree with me that it only irritated Sandra Bland more. But maybe that was the purpose: listen next to the state trooper "asking" her to put out her cigarette, and especially his tone of voice. Contact Basemetal here 15:56, 24 July 2015 (UTC)[reply]
Not for nothing, but ... it wasn't going to take "much" to get her "irritated". It was quite obvious that she was "itching for a fight". In my opinion. Joseph A. Spadaro (talk) 18:36, 25 July 2015 (UTC)[reply]
I would partially agree with your statement joseph spadaro. While it seems true that she had an immediate bias against the officer and even if she was 'itching for a (verbal) fight" that still doesn't excuse the fact that the officer is a total buffoon and can't behave appropriately. He went from ASKING (not demanding) that she put out the cigarette (which is dumb to begin with) to blowing up on her 'get out of the car' 'get out of the car' yelling and hollering. Void burn (talk) 18:48, 25 July 2015 (UTC)[reply]
Yes, but ... (1) the request to put out the cigarette was not "dumb"; it was legal, appropriate, and standard police protocol; and (2) the police stop has absolutely nothing to do with her death, three days later. Everyone may want to armchair quarterback how good/bad the police acted during the traffic stop. And all of that has zero relevance to the jail death. In my opinion. Thirdly, a great way to get respect is to give respect; in other words, it's a two-way street. Some people simply want to be respected (to not be "dissed"), yet feel entitled that they themselves can exhibit all sorts of disrespect to the other party. Joseph A. Spadaro (talk) 19:11, 25 July 2015 (UTC)[reply]
The officer may be within his legal authority to demand that she put out the cigarette, but it very much seems when observed in context (and listening to the quality of the officer's tone and emphasis) that the officer issued that order specifically as a result of Ms. Bland's accusatory tone. The emphasis certainly seems to suggest to me that it was a tit-for-tat response to her own short temper. On the other hand, it is not outside the realm of possibility that he simply issued that order because he foresaw the possibility that the stop may become more complicated than it at first seemed, and didn't want it to be a factor, and that the stress he puts on the command is simply a result of his own raised state of tension. Really it is impossible to say with any certainty. Certainly there's a point within a couple of minutes later where he clearly loses any consistency with prescribed protocol; "I will light you up!" is just not something that is supposed to come out of any officer's mouth outside an 80's buddy cop flick. However, as to the specific question of the OP... (see bellow)Snow let's rap 23:36, 25 July 2015 (UTC)[reply]

It's worth noting that he asked the same manner of "You ok?" question of the person he stopped immediately previous. It may very well have been a part of his training or otherwise a standard part of his approach. However, his use of the phrase is slightly contextually different, and the (phonetic) emphasis in the second case strikes me as more confrontational than the first, which is accompanied by a disarming little laugh. It's difficult to parse and to understand what he hoped to gain from asking the question (and the follow-up) if his motivation for asking was that he already could sense that she was irritated. But note that this is not a reaction that is utilized just by people who want to make someone back down from their display of irritation; people of all stripes have a natural inclination to ask that sort of question when the sense unspoken hostility. Most people will be very uncomfortable in the presence of unspoken antipathy and will seek to clarify it, regardless of whether they just want to know the other party's state mind or have already guessed it and want it to be verbally acknowledged so that they can respond to it in a way that assets their control or perspective on the matter. Snow let's rap 23:36, 25 July 2015 (UTC)[reply]

Tokyo green belt

Does Tokyo have a green belt? I can't find a single map showing one. 176.250.110.177 (talk) 21:12, 23 July 2015 (UTC)[reply]

I can't find a direct answer to your question, but This document seems to be useful in determining general urban land-use policies in Japan. --Jayron32 21:18, 23 July 2015 (UTC)[reply]
That very useful link refers to zones called "urbanization control areas" (UCAs), which appear to be broadly comparable with green belts (as the term is understood in the UK - nothing to do with public parks, etc.). There is some more information about what is allowable within urbanization control areas here, here, etc. Ghmyrtle (talk) 08:08, 24 July 2015 (UTC)[reply]
...and this document may give you the information you need about Tokyo specifically. Ghmyrtle (talk) 08:15, 24 July 2015 (UTC)[reply]
A look at Google Maps shows a sprinkling of small isolated parks, nothing comparable, say, to Manhattan's Central Park. μηδείς (talk) 21:42, 23 July 2015 (UTC)[reply]
Green belt, eh? No answers here, just linking. May as well do Green belt (United Kingdom), too. InedibleHulk (talk) 01:41, July 24, 2015 (UTC)
Ueno Park, the largest park in Tokyo, is 54 ha (according to WP). (Central Park is 341 ha). For other parks see Parks and gardens in Tokyo. There's a lot of green around the imperial palace. About the same size as Central Park. But I don't think that is accessible to the public. Not far from there you've got Hibiya Park, about 16 ha. Incidentally, can a park in the middle of the city (like Central Park) really be what is meant by a "green belt"? I thought a green belt was supposed to be something that surrounds a city. Contact Basemetal here 16:29, 24 July 2015 (UTC)[reply]
Agreed: a "green belt" is a protected rural zone surrounding a conurbation to prevent urban sprawl - see the articles linked by InedibleHulk above. Thus the question is about green space around Tokyo, rather than green space within Tokyo. Alansplodge (talk) 18:28, 24 July 2015 (UTC)[reply]
NYC does have such parks, as Pelham Bay Park and Van Courtland Park as well as Forest Park (Queens) and Highland Park (Brooklyn). Although they may not necessarily fall on the current outside borders of NYC itself, when parks like Central Park were designed they were indeed on the outskirts of inhabited areas. Unless Mayor DeBlasio bombs Yonkers, a contemporary greenbelt is unlikely. Nevertheless, NYC is much greener than Tokyo. μηδείς (talk) 21:09, 24 July 2015 (UTC)[reply]
A greenbelt is not a park or a series of parks, but any rural land (agricultural, forestry etc) which is protected from urban development by planning legislation. Alansplodge (talk) 13:57, 25 July 2015 (UTC)[reply]
Yeah... we do have to be careful with terminology here... it may be helpful to note the subtle distinction between a Green belt and a Greenway... as has already been pointed out, New York City has the Brooklyn–Queens Greenway (a series of interconnected parks)... but that is not a Green Belt. Blueboar (talk) 15:33, 25 July 2015 (UTC)[reply]

July 24

Before the Copyright Act of 1976, were registration and deposit mandatory? I'm holding a 1940-published book that's in the public domain (it doesn't appear in the renewals database), but I'm wondering if it were ever copyrighted. "All rights reserved; copyright 1940" are given in the normal place, along with the author's name, but I can't find evidence that it was registered. In such a case, did it pass into the public domain in 1940 or in 1968? Nyttend (talk) 02:24, 24 July 2015 (UTC)[reply]

My reading of Cornell's excellent copyright table would suggest that a work first published in the United States with a copyright notice did not require registration to be protected, at least after 1923. Someguy1221 (talk) 03:17, 24 July 2015 (UTC)[reply]

Wire transfer with different beneficiary bank and receiving bank

For some incoming wire transfer instruction (e.g. [15]) I see the recipient listing a "beneficiary bank" and a separate "receiving bank" where the two banks are completely different entities and could even be in separate countries. Is this common around the world? Or is this just a North American thing? Most banks can't seem to hand this type of situation.

For the "normal" case where there's only one bank listed in the wire transfer instruction, would that bank be considered the "beneficiary bank" or the "receiving bank"? My other car is a cadr (talk) 03:39, 24 July 2015 (UTC)[reply]

This one[16] is even crazier with three different banks, two in Canada and one in the US. My other car is a cadr (talk) 04:10, 24 July 2015 (UTC)[reply]

See Correspondent account - it's not at all uncommon to have multiple institutions involved in international transfers (both of your examples involve the US and Canada). In your "normal" case, the answer is both - since there's no other insitution involved, then the insitution receiving the funds is also the beneficiary of the funds. Most wire forms will probably label that bank as the beneficiary in this situation, but both terms would apply. Also, I disagree with your characterization that most banks can't handle these situations. It may be uncommon for most staff in a typical retail bank branch to encounter, but the vast majority of banks have the capability of handling complex wires like this. I manage a retail branch for a regional American bank - such a wire landing on my desk would involve a phone call to the wire department and reading up on some procedures, but it could definitely be done. 12.71.77.125 (talk) 13:36, 24 July 2015 (UTC)[reply]

Ethiopia: a landlocked nation

Does Ethiopia regard its lack of harbors, ports and coastline to be an issue similar to Bolivia? None of its border disputes with Eritrea are related to coastline territory. Is the nation perfectly fine without a coastline? --68.116.114.141 (talk) 17:00, 24 July 2015 (UTC)[reply]

I don't think it is "perfectly fine" without a coastline. At least judging by this or this. If you Google "Ethiopia landlocked problem" you'll get more stuff. Here you can see for example views expressed in a forum. Contact Basemetal here 17:13, 24 July 2015 (UTC)[reply]
Even before Eritrea achieved independence, Djibouti was the main cargo port for Ethiopia. So the issue is not solely linked to bilateral difficulties with Eritrea. --Xuxl (talk) 10:24, 25 July 2015 (UTC)[reply]

By-election results

Are there any online sources for UK Parliamentary by-election results?

JASpencer (talk) 18:41, 24 July 2015 (UTC)[reply]

Wikipedia? AndrewWTaylor (talk) 19:17, 24 July 2015 (UTC)[reply]
Also List of United Kingdom by-elections (1979–present). There are a lot of resources available at www.parliament.uk By-elections, of which UK Election Statistics: 1918-2012 seems to be the most pertinent. Alansplodge (talk) 20:43, 24 July 2015 (UTC)[reply]

Thank you for the responses. The issue I'm having is that I'm writing articles about by-elections and there are a lot of by-election results that are uncited which I'd like to confirm and cite. So are there any sites that have these results? 08:54, 25 July 2015 (UTC)

Does this provide you with what you're looking for?--Phil Holmes (talk) 09:41, 25 July 2015 (UTC)[reply]
Thank you for that, but I'm looking for individual breakdowns like this wayback page. JASpencer (talk) 09:28, 26 July 2015 (UTC)[reply]
I've created a list from 1945 to 79 here and here for 79 to 97. JASpencer (talk) 21:56, 26 July 2015 (UTC)[reply]

At What Point Does a Treaty Become Invalid or Void Because It's Terms Are Secret or Unwritten?

I had always assumed that the Law of Treaties was a body of international law containing well-understood and clearly-defined requirements for the formation of valid treaties. Recent controversy over the treaty negotiated between the United States of America and its allies with the Islamic Republic of Iran have led me to ask:

1.) Is it possible for a treaty to be valid if it contains provisions not fully disclosed or understood by all parties to the treaty? 2.) Is it fundamentally necessary that all portions of a treaty be in writing?Honeyman2010 (talk) 23:31, 24 July 2015 (UTC)[reply]

(1) First, a point of clarification: the Joint Comprehensive Plan of Action is not a treaty in the U.S. law sense of the word. Not all international agreements are treaties in the U.S. constitutional law sense. The JCPOA is an executive agreement. (See testimony of David Albright, President of the Institute for Science and International Security (ISIS)).
(2) See our (woefully unreferenced) article on secret treaty. Prior to the First World War, such treaties were relatively common. (Indeed, such treaties were one of the causes of World War I).
One of Woodrow Wilson's Fourteen Points (1918) was the abolition of secret treaties, and generally the institution of public, open diplomacy. This eventually came to pass with Article 18 of the Covenant of the League of Nations, which required members of the League to register agreements with the League secretariat: "Every treaty or international engagement entered into hereafter by any Member of the League shall be forthwith registered with the Secretariat and shall as soon as possible be published by it. No such treaty or international engagement shall be binding until so registered."
This rule is preserved in the present day via the United Nations - see Article 102 of the United Nations Charter: 1. Every treaty and every international agreement entered into by any Member of the United Nations after the present Charter comes into force shall as soon as possible be registered with the Secretariat and published by it. 2. No party to any such treaty or international agreement which has not been registered in accordance with the provisions of paragraph 1 of this Article may invoke that treaty or agreement before any organ of the United Nations. The Vienna Convention on the Law of Treaties also entrenches the registration/depository system.
(3) As to whether an oral agreement can be formed and be as binding as a written agreement: theoretically, yes, see Article 3 of the Vienna Convention on the Law of Treaties: The fact that the present Convention does not apply to international agreements concluded between States and other subjects of international law or between such other subjects of international law, or to international agreements not in written form, shall not affect: (a) the legal force of such agreements... So foreign minister X could make a binding oral agreement with foreign minister Y. In reality, all serious agreements intended to be binding will be reduced to a formal writing, pored over by diplomats. Even a preliminary agreement is reduced to writing (see Modus vivendi).
(4) As to situations when a treaty might become void or invalid: see Vienna Convention on the Law of Treaties articles 46-53, which outline such situations. Article 48, on error, might be what you're thinking of. Neutralitytalk 03:26, 25 July 2015 (UTC)[reply]
Who will step up to improve the "woefully unreferenced" article on secret treaty? -- Paulscrawl (talk) 05:19, 25 July 2015 (UTC)[reply]
Excellent answer from Neutrality. But we do have an article on the Ihlen Declaration, where the World Court ruled that foreign minister X did make a binding oral agreement with foreign minister Y.John Z (talk) 05:32, 26 July 2015 (UTC)[reply]

July 25

When did The Adventures of Rupert Bear run?

Hi all, I'm having a dickens of a time tracking down information on The Adventures of Rupert Bear, a British children's television series that may have run circa 1967-1977. The article is unsourced, I think at the very least knowing when the series started would be of tremendous help, but any other info that could be corroborated, for instance the network it ran on and number of episodes would be fantastic. Don't mean to dump this on the Ref Desk, but I'm not sure how else to get this info. Danke! Cyphoidbomb (talk) 17:50, 25 July 2015 (UTC)[reply]

IMDB (which is not a fully reliable source for the purposes of citing for Wikipedia articles, but may be a good starting point for your research) says that it ran in first run episodes from 1970-1972. It also appears there was a revival called just Rupert Bear that ran from 2006-2007. Hope that helps! --Jayron32 18:04, 25 July 2015 (UTC)[reply]
h2g2 ok for a source? [17]. —eric 18:34, 25 July 2015 (UTC)[reply]

Israel Shahak quotes?

It has been shown at times that Israel Shahak was not averse to fabricating his quotes (see the WP article), so I wonder if anyone can verify the following two alleged quotes he inserted in two books of his.

  1. This article by Robert Fisk from the English Independant (Wednesday 03 December 1997) has the following: He [Shahak] quotes from an official exhortation to religious Jewish soldiers about Gentiles, published by the Israeli army's Central Region Command in which the chief chaplain writes: "When our forces come across civilians during a war or in hot pursuit or a raid, so long as there is no certainty that those civilians are incapable of harming our forces, then according to the Halakhah (the legal system of classical Judaism) they may and even should be killed ... In no circumstances should an Arab be trusted, even if he makes an impression of being civilised ... In war, when our forces storm the enemy, they are allowed and even enjoined by the Halakhah to kill even good civilians, that is, civilians who are ostensibly good." Robert Fisk is quoting from Shahak's book Jewish History, Jewish Religion: The Weight of Three Thousand Years: Pluto Press, London, 1994. Can anyone verify that quote in Shahak's book and see if he gives any reference to any actual document?
  2. This article on web site Loonwatch.com by someone writing under the name of "Danios" has the following: Prof. Israel Shahak, an Israeli human rights activist, documented the background for this racist religious dogma in his book Jewish Fundamentalism in Israel. For example, he quotes Rabbi Abraham Kook, largely considered “the ultimate father figure” of Religious Zionism, who stated that “the difference between a Jewish soul and the souls of non-Jews ... is greater and deeper than the difference between a human soul and the souls of cattle.” As you can see the quotation is referred to Shahak's book (co-authored with Norton Mezvinsky) Jewish Fundamentalism in Israel: Pluto Press, London (there are two editions, 1999 and 2004, but the guy doesn't say which one he is citing). Again, can anyone verify that quote in Shahak's book and see if he refers to any place in Abraham Isaac Kook's work where this quote is supposed to be from?

Thanks. Contact Basemetal here 19:00, 25 July 2015 (UTC)[reply]

Was this Israeli soldier quoting the Bible?

This Google book preview won't let me see the footnote (footnote 31) to the following quote (line 6): It's not clear to me what a Hebrew soldier is doing so far from home. I have the feeling the what a Hebrew soldier is doing so far from home part echoes some passage in the Bible. Do you recognize such a passage in the Bible? Can you read footnote 31? If yes, what does it say? Thanks. Contact Basemetal here 19:34, 25 July 2015 (UTC)[reply]

The footnotes are viewable on this Amazon preview (p. 246) and says: 31. Sandro Contento, "Israel Bans TV Crews from Filming Army Raids", Toronto Star, 24 March 2002. So just an attribution. It's not a well-known Biblical quote as far as I know. Alansplodge (talk) 20:24, 25 July 2015 (UTC)[reply]

About a scene in Eisenstein "Ivan the Terrible" Part 2?

I have a couple of questions about this scene (direct link to the scene 32 minutes 42 seconds into the movie) in Sergei Eisenstein's movie Ivan the Terrible Part 2 (it's the first scene in the cathedral, with the song in the fiery furnace, after the book of Daniel; the greatest in the whole movie but we're not supposed to express opinions here; no wonder Stalin didn't like this movie; if the time offset doesn't work you can try this alternate link or if neither time offset format works it's 32 minutes 42 seconds into the movie).

  1. Is the music of this particular scene (more particularly the song that the children are singing in the fiery furnace) a traditional Orthodox church melody or is it an original composition by Sergei Prokofiev?
  2. Do these kinds of religious plays still take place in Russian Orthodox churches? If yes, for what feast(s)?
  3. Does the kind of clowns who play the Chaldeans in that religious play have a name in Russian?

Thanks. Contact Basemetal here 20:29, 25 July 2015 (UTC)[reply]

PS: Slightly better audio here or here (or, manually, 34 minutes 22 seconds into the movie). Contact Basemetal here 20:58, 25 July 2015 (UTC)[reply]

This article says "[t]he Furnace Play is an ancient Russian liturgical drama which was performed in the Uspensky (Dormition) cathedral in the Moscow Kremlin on the Sunday of the Holy Fathers, a week before Christmas", and also that the music is original to the film. Admittedly that doesn't exactly answer your first two questions, since it may not still be performed and the music may be based on a traditional tune. -- BenRG (talk) 03:36, 26 July 2015 (UTC)[reply]
Thanks Ben. This is already valuable. Since the words of the song were written by Eisenstein I'd be willing to bet it is not based on a traditional tune because it would be an amazing coincidence if the prosody of the words written by Eisenstein matched that of some traditional Orthodox song. This said it is clear Prokofiev made it sound like a traditional Orthodox hymn. Nitpicking: The Pravda says the Sunday of the Fathers falls in the week before Christmas but that's only if you take weeks to start on Mondays. Maybe that's how it works in Russia but that's not the traditional Jewish and Christian position. (Or if Christmas falls on Sunday) Contact Basemetal here 15:39, 26 July 2015 (UTC)[reply]

The UK-Parliment is a perfect o imperfect bicameral?

Sorry, i anticipe i'm italian. I don't understand if the parliment of the United Kingdom is perfect or imperfect bicameral. The maine problem is about house of lords, i don't understand if it can fall down the govern or not, and how this house is elected. Because i know what the ereditariety of the seats, but other? How are they elected or nominated? How time do they stay at them position? I see the colors of the political parties on the page, so there is a partitic domination in this house. What is the derivation of this partitic control in this house. Thank you anticipatly. --79.54.130.144 (talk) 20:48, 25 July 2015 (UTC)[reply]

As a rule, nothing in the UK political system is "perfect" - it's not designed, but slowly evolved, with plenty of quaint and not-so-quaint quirks. Our article is at House of Lords. Members are appointed by the Queen, but she follows the advice of the House of Lords Appointments Commission. The influence of the House of Lords is limited nowadays. It cannot topple the government, and it can only delay bills. Strictly the government does not depend on the House of Commons, either, but again tradition dictates that it resigns if it loses support there. --Stephan Schulz (talk) 21:13, 25 July 2015 (UTC)[reply]
Just to fill in a couple of details for the OP: The House of Lords can't veto a Government bill, under the Parliament Act 1911. There is no upper age limit or fixed term of service - members of the house can stay there until they die, resign, or are expelled. The distribution between the parties was set up by the House of Lords Act 1999 (which removed the automatic right of hereditary peers to sit in the House), and is maintained by the Appointments Commission. Before 1999, the Conservative party had a very large majority, mainly made up of hereditary peers. Tevildo (talk) 21:19, 25 July 2015 (UTC)[reply]
You may also be interested in Reform of the House of Lords. Nearly everyone thinks it should be replaced by something better but nobody knows what. Alansplodge (talk) 21:26, 25 July 2015 (UTC)[reply]
Why not just get rid of it then? Contact Basemetal here 21:49, 25 July 2015 (UTC)[reply]
It's the UK. They never give up anything for good. You never know when a trained group of longbow men might come in handy! --Stephan Schulz (talk) 22:20, 25 July 2015 (UTC)[reply]
Quite right - see Royal Company of Archers. Alansplodge (talk) 23:27, 25 July 2015 (UTC)[reply]
Some people support that idea (including 22% of Britons in a 2010 poll and 163 MPs in a 2007 vote). Neutralitytalk 22:01, 25 July 2015 (UTC)[reply]
Because of the principle of Supremacy of Parliament, many think it's sensible to retain a second chamber as some sort of filter for poor legislation that might be rushed through the Commons. Exactly what form that should take is debated; there is little enthusiasm for another elected body. Alansplodge (talk) 23:24, 25 July 2015 (UTC)[reply]
I know some have floated the idea of a reformed House of Lords as a "House of Experts" - basically, making it all-crossbenchers, all appointed in nonpartisan fashion. That sounds like a reasonably sensible idea. Neutralitytalk 01:49, 26 July 2015 (UTC)[reply]
Meritocracy and technocracy are variants on this idea. --Jayron32 02:00, 26 July 2015 (UTC)[reply]
"Perfect" means something specific in this context. We don't have an entry for it, but the article on the Italian Parliament defines a perfect bicameral system as one where both chambers have identical rights and powers. By this definition, the British system is very definitely "imperfect". Rojomoke (talk) 12:19, 26 July 2015 (UTC)[reply]

Um -- such a "perfect bicameral" legislature would be odd -- if the two parts are identical in "rights and powers" then there is no reason for the second body. In most places with bicameral legislatures, the second body is one for deliberation more than for initiating legislation. "Bicameral" just means "having two chambers" - that is all it means. A "perfect" one would be one that has precisely two chambers, no matter what any Wikipedia article in Italy states <g>. Collect (talk) 12:37, 26 July 2015 (UTC)[reply]

As distinct from approximately two chambers, maybe? How does rounding work in the case of two and a half chambers? -- Jack of Oz [pleasantries] 21:44, 26 July 2015 (UTC)[reply]

When a person dies, are you supposed to notify the credit reporting agencies?

When a person dies, are you supposed to notify the credit reporting agencies? This is for the USA. Thanks. 2602:252:D13:6D70:A597:AD3E:4899:DD6D (talk) 21:17, 25 July 2015 (UTC)[reply]

The Probate laws of each state can vary, but in general this is likely to be one of the tasks handled by the Executor. ←Baseball Bugs What's up, Doc? carrots21:44, 25 July 2015 (UTC)[reply]
It is a good idea to do so, if you are the surviving spouse of the decedent or the personal representative of the estate, in order to minimize the risk of identity theft. See here (from Experian) and here (a guide from the Illinois law firm of Meyer Capel). Neutralitytalk 21:51, 25 July 2015 (UTC)[reply]
OK. But, I am missing something here. In all honesty, who would really (practically) care if the deceased person had their identity stolen? 32.209.54.215 (talk) 01:22, 26 July 2015 (UTC)[reply]
If an identity thief gets control of a decedent's identity, it (1) the beneficiaries would care - because the thief might incur debts that the estate might have to pay (this is complicated and context-dependent), thus diminishing the amount that the beneficiaries of the estate can collect; (2) the personal representative (and the estate's attorney, if there is one) would care - because it can cause a big headache for the settlement of the estate's affairs; (3) the government would care - if the identity thief fraudulently collects benefits using the decedent's identity; (4) a pension fund would care - for the same reason; and (5) lenders and creditors would care - because identity theft is a cost to them.
See also Ghosting (identity theft). Neutralitytalk 01:38, 26 July 2015 (UTC)[reply]
Yes, I can see why all those people would "theoretically" care. But, it will be easy to ascertain with definitive proof that the person in question died (or did not die). Once the death is "proven", aren't all those "identity theft" issues resolved and returned to status quo? In other words, any "impact" on the estate is negated/neutralized; any pension payments would be stopped; any debts would be written off; etc. Joseph A. Spadaro (talk) 04:36, 26 July 2015 (UTC)[reply]
How do you figure the debts would be written off? The money is still owed, and depending on exactly how a given state's laws work, I would expect it to be paid by the estate. ←Baseball Bugs What's up, Doc? carrots04:53, 26 July 2015 (UTC)[reply]
@Baseball Bugs: I don't understand your post/your question. Am I not correct to assume that a person is not liable for a debt that was obtained by fraud? If a person (through identity theft) ran up a huge credit card bill with the credit card of a deceased person, I cannot imagine that the deceased person (or his estate) is liable for a debt that is 100% proven to be fraudulent (i.e., there is no dispute that the decedent is, in fact, dead; and could not have possibly charged those debts on the credit card). I assumed that the credit card/bank writes that off, when the fraud is discovered. No? Joseph A. Spadaro (talk) 06:31, 26 July 2015 (UTC)[reply]
A good reference:
Credit Bureaus
To head off identify theft, experts recommend reporting the death to major credit-reporting agencies. Once a “deceased alert” is placed in the file, any requests for credit will be recognized as fraudulent and rejected. The big three agencies are Equifax (www.equifax.com, 800-525-6285), Transunion (www.transunion.com, 800-680-7289), and Experian (www.experiancom,888-397-3742).
Randolph, Mary (2008). The Executor's Guide: Settling a Loved One's Estate or Trust (3rd. ed.). Berkeley, CA: Nolo. p. 67. ISBN 978-1-4133-0655-2.
-- Paulscrawl (talk) 06:16, 26 July 2015 (UTC)[reply]

history of the current european immigration laws.

Please do not publicize my Ip. I am a contributor to Wikipedia and probably have an account. My questions is this: how do I go about finding info in Wikipedia as to the history of the current European countries immigration laws?

Thank you,

Ilona Proska

— Preceding unsigned comment added by 2601:243:1:9385:6040:9d25:aaed:dc37 (talkcontribs)

To clarify the message you were shown before posting without logging in:
"You are not logged in. Your IP address will be publicly visible if you make any edits."
If you have an account but post without logging in, your IP address is automatically logged. See WP:IP.
If ypu need help to recover your password &/or username, check Help:Reset_password
On your question, try Immigration law. -- Paulscrawl (talk) 04:54, 26 July 2015 (UTC)[reply]

Is Wikipedia English or American?

I know the language of Wikipedia is supposed to be British-English, not American-English. My question is not about language, it is about, how should I put it, which mentality, which way of thinking, which standards. Thanks. Akseli9 (talk) 11:32, 26 July 2015 (UTC)[reply]

Confusingly and irritatingly, it is both, depending on the article subject and/or who originally wrote it. See WP:ENGVAR. Adam Bishop (talk) 11:39, 26 July 2015 (UTC)[reply]
Who says "the language of Wikipedia is supposed to be British-English, not American-English"....? "The English Wikipedia prefers no major national variety of the language over any other." Ghmyrtle (talk) 11:42, 26 July 2015 (UTC)[reply]
... and the fundamental standards of Wikipedia are the five pillars. I don't think these are particularly British or American. How could you tell ? Gandalf61 (talk) 11:56, 26 July 2015 (UTC)[reply]
In some respects, it's actually French. The "Wikipedia mentality" (openness, collaboration, pursuit of knowledge, objectivism, the view that the common people can contribute, etc.) derives in large part from Enlightenment philosophy, which comes from all across Western Europe, but it often identified most closely with French thinkers. More directly, it stems from similar views as the open-source movement and free software movement (both worldwide, but associated with American "founders") - but applied to knowledge, rather than software. Even more directly, Wikipedia was created in America by Americans (Jimmy Wales and Larry Sanger), but with the intent of a global reach. -- 160.129.138.186 (talk) 17:45, 26 July 2015 (UTC)[reply]

American neutrality or common mistake?

Thank you all for your useful comments and links. As I said, my question was not about the language and British-English (thanks again for the links). I have a question about the article July 24, and in general, about the way we should handle notable people's place of birth, nationality, and appurtenance (where people belong, where people feel they belong). In such article as the July 24 article, I can find many examples of what I would call a typically American way of thinking people's nationality, people's appurtenance. My question is, is it just a mistake by one random contributor, or is it more about an American logics of assigning nationalities and appurtenances?

There would then be a subsidiary question about the choice of forgetting "West-" and keeping only "German", for people whose notability exists only within the cold war period when there were two distinct Germanies. Makes me recall also the Yugoslavian example, which is sometimes wrongly replaced by "Slovenian" or "Serbian" or such.

Thanks. Akseli9 (talk) 09:10, 27 July 2015 (UTC)[reply]

I'm sorry Akseli9, I don't understand the question. Specifically: I don't know what you mean by "American" or "British" mentalities or standards. What is a "typically American way of thinking people's nationality, people's appurtenance", and which ones do you think might be "mistakes"? Iapetus (talk) 12:13, 27 July 2015 (UTC)[reply]
I'm sorry for being so unable to make myself clearer. With the help of your focussed questions we shall eventually reach the point. I think a "typically American way of thinking people's nationality, people's appurtenance", is to view it the way they view it in American countries (especially the US but I'm thinking also of Brasil). In such article as the July 24 article, you find a list of people who sometimes are just "American", and sometimes can be "South-African American", or "Polish-American", etc. You find also some Germans who according to precedent contributor was not German but "West-German". You find also that interesting case of a French who was born in French Algeria (before 1962, thus not in Algeria), who according to some contributors, becomes an "Algerian-French" or an "Algerian-born". My focussing on American logics comes from the fact that American culture is so omnipresent in our very lives and indeed thoughts, I was wondering if this way of assigning nationalities to people, was perhaps coming from this constant americanization of ours? One obvious mistake in the July 24 article or in her own article, was to consider French alpinist Catherine Destivelle as an "Algerian-born" or as an "Algerian-French", but I'm wondering whether there could be a lot of similar mistakes in the entire encyclopedy? Akseli9 (talk) 12:40, 27 July 2015 (UTC)[reply]
Please give an example or two of where someone is described as "American" and you think it is a mistake. Regarding your example, it is not a "mistake" to describe someone as French-Algerian rather than French. This is a matter of opinion and Wikipedia style, and it is not as clear-cut as you seem to believe it is. Furthermore, you seem to think that describing someone's nationality in a particular way is somehow a distinctively American thing to do, which is also incorrect. --Viennese Waltz 12:51, 27 July 2015 (UTC)[reply]

Albert Einstein long cat quote

"You see, wire telegraph is a kind of a very, very long cat. You pull his tail in New York and his head is meowing in Los Angeles. Do you understand this? And radio operates exactly the same way: you send signals here, they receive them there. The only difference is that there is no cat." -Albert Einstein [18][19]

Did Einstein really say this? If so, is there an authoritative source proving it?

I have a feeling this is a recently invented quote that's mis-attributed to him, since it sounds suspiciously like the the mash-up of two memes: longcat and series of tubes. My other car is a cadr (talk) 12:08, 26 July 2015 (UTC)[reply]

[20] gives an anteceding quote a date of 1866. Result: Not Einstein. Collect (talk) 12:29, 26 July 2015 (UTC)[reply]
Thanks.My other car is a cadr (talk) 14:14, 26 July 2015 (UTC)[reply]
Resolved
  • Not so fast! The 1866 version was only about the telegraph, not radio. The punch line that with radio "the dog is imaginary" was not added, according to the source Collect cited, until 1917. Of course that doesn't mean that Einstein said it; but it does mean that we can't rule it out on the grounds of the date alone: his career as a physicist was well established by then. However, according to the same source, the 1917 version was originally rendered in "heavy dialect", and Einstein's name was not mentioned in connection with it or with a 1924 version that's close to the one we were asked about. I'd say that makes it extremely unlikely that it was him. --65.94.50.73 (talk) 21:00, 26 July 2015 (UTC)[reply]
It's not in The New Quotable Einstein. That doesn't prove he didn't say it, and not everything he ever said is in there, but it's the sort of quote likely to get a guernsey if it were authentic. (The internet has spawned a whole new industry of misattributed quotations, some of which are patently absurd and obviously made up, but which then get copied as gospel truth and enter the belief systems of people who don't know any better, who often then strenuously defend the veracity of the attributions because first impressions last.) -- Jack of Oz [pleasantries] 21:38, 26 July 2015 (UTC)[reply]
Schrödinger: "Hold on there, Albie. Maybe there's a cat, and maybe there isn't. Did you look?" Clarityfiend (talk) 22:34, 26 July 2015 (UTC)[reply]

Oldest named animal

Who was the oldest historically verifiable animal (not myths or gods) given a name? An animal older than Bucephalus. Are there any records/documents in which a pet dog or cat or warhorse is given a name/refer to by a name dating prior to the 4th century BC?--KAVEBEAR (talk) 02:00, 27 July 2015 (UTC)[reply]

[21] People in ancient Egypt gave names to their dogs - as evidenced by names on their collars. Say 3000 BCE or so. Cats appear to have been generally called "Cat" ("Mau" or "Miw") or the like - but since they were gods, giving one a demeaning name would have been problematic <g>. Collect (talk) 02:15, 27 July 2015 (UTC)[reply]

Christian missionaries involved in Mangareva and the Gambier Islands

I'm trying to understand Catholic missionaries that were active in Mangareva and the Gambier Islands in the 19th century besides Honoré Laval. I know of Francois Caret, Columba Murphy, Cyprien Liausu, Étienne Jérôme Rouchouze, and Louis Désiré Maigret. Can somebody help me out with finding names of additional missionaries in the islands at the time?--KAVEBEAR (talk) 02:36, 27 July 2015 (UTC)[reply]

Why would Congress want an even number of justices on the U.S. Supreme Court?

I was reading the introductory paragraphs in this article: List of Justices of the Supreme Court of the United States by court composition. It seems that there were many times during which our U.S. Supreme Court had an even number of seats. Why would the legislators desire that? I always assumed that, in situations such as this, an odd number of voters is preferable (to avoid ties and "split" courts). An even amount of "voters" (justices) makes no sense. What would be the rationale or the thinking back then? Thanks. Joseph A. Spadaro (talk) 05:33, 27 July 2015 (UTC)[reply]

http://www.thecrimson.com/article/2006/2/5/and-then-there-were-eight-the/ --KAVEBEAR (talk) 05:59, 27 July 2015 (UTC)[reply]
Very interesting article. Thanks! Joseph A. Spadaro (talk) 07:27, 27 July 2015 (UTC)[reply]
A tie vote, as I'm sure you're aware, affirms the lower court's decision. I suppose that there could be some political edge to a court that would have a predisposition to affirm, but I'm not sure what it is. GregJackP Boomer! 06:20, 27 July 2015 (UTC)[reply]
Excellent question. One possible political edge/advantage might be judicial stability? Maintaining the status quo? That is, cases getting affirmed (i.e., settled) more often than being "disturbed" (changed). I guess that can be an advantage, though not necessarily a political one. Joseph A. Spadaro (talk) 07:35, 27 July 2015 (UTC)[reply]
(EC):Technically "ties" in the Supreme Court aren't considered ties. A "tie" in the Supreme Court is treated the same as "win" for the defendant (the lower court's decision remains in effect). SCOTUS tradition holds that ""no affirmative action can be had in a cause where the judges are equally divided in opinion as to the judgment to be rendered or order to be made."[22] This happens more often that you'd think, even with an odd number of appointed Justices, mostly due to a Justice recusing him/herself, but also due to the President not being able to get his nominee confirmed in time to hear cases. Even with today's full nine Justice panel, a 5-4 (or sometimes even a 6-3) ruling is still often considered a "split court", especially if one of the majority issues a concurring opinion that differs substantially from the majority opinion. And while that doesn't change the outcome of the case, it considerably weakens it and can leave a lot of issues open for scrutiny in later cases. So, having an odd number of seats isn't as important as it may seem at first blush.
No, a tie is not the same as a win for the respondent (the side that won in the lower court). See Supreme Court of the United States#Decision. It's the same as regards the outcome that particular case, but it's deemed not to establish a precedent; very likely a similar case will be appealed to settle the question as soon as there are 9 justices available. --65.94.50.73 (talk) 07:34, 27 July 2015 (UTC)[reply]
Excellent point. That is a very subtle – but critical – distinction. Joseph A. Spadaro (talk) 07:39, 27 July 2015 (UTC)[reply]
As to your question, I believe the thinking behind having an even number of seats was that a 4-2 ruling, for example, was clear-cut and made more of a statement than would a 4-3 ruling. Also, as our article Supreme Court of the United States says near the top: "Because the full Court had only six members, every decision that it made by a majority was also made by two-thirds (voting four to two)." Also in that same article, it points out that the Court grew with the nation, adding Justices for each new judicial circuit created. "As the nation's boundaries grew, Congress added justices to correspond with the growing number of judicial circuits: seven in 1807, nine in 1837, and ten in 1863." The number of Justices wasn't fixed at nine until 1869. Then there was FDR's attempt to pack the Court, but that's a different story.--William Thweatt TalkContribs 06:22, 27 July 2015 (UTC)[reply]
Thanks, all. I just read the article linked above (And Then There Were Eight). It states: "Alternately, a 4-4 tie would send the case back to lower courts—either to the states or the federal circuits." So, that statement is incorrect? Joseph A. Spadaro (talk) 07:31, 27 July 2015 (UTC)[reply]
If you read the sentence in context, it's clear that the author means "sends the case back unchanged", i.e. lets the lower courts decision stand. The phrasing is a bit unfortunate, but its not strictly wrong. --Stephan Schulz (talk) 07:50, 27 July 2015 (UTC)[reply]
Yes, I see your point. But, it doesn't really "send anything back", correct? It just allows the lower decision to stand. Or does it "send it back" to the lower court, and then the lower court has to "do" something? (Like "affirm" its last decision?) Or "enter" its last decision as a final judgment? Is there anything "physical" or "affirmative" that the lower court has to "do", once the Supreme Court sends it back? Thanks. Joseph A. Spadaro (talk) 08:19, 27 July 2015 (UTC)[reply]

Mute turns verbose

I read once about someone who went on to become famous (possibly a philosopher) who was mute as a child until aged four or five he came out with, as his first words, a long and complex sentence, possibly witty, which was aimed at a servant and possibly began with the word, "Madam". Can anyone help from these slender clues? --Dweller (talk) 08:53, 27 July 2015 (UTC)[reply]

I think you are talking about Thomas Babington Macaulay. I don't think he was mute before, but on having had hot coffee spilt on him at the age of three, is said to have announced: "Thank you, Madam, the agony is sensibly abated". The story is recounted here, but I observe it is not in our article or in wikiquote:Thomas Babington Macauley. --ColinFine (talk) 09:26, 27 July 2015 (UTC)[reply]
That looks right, thanks. Wonder where I got the mute bit from? --Dweller (talk) 09:32, 27 July 2015 (UTC)[reply]
In the version I read, he had not previously spoken up to that point; there may have been confusion in variant accounts between "mute" meaning "not speaking" and "mute" meaning "unable to speak." {The poster formerly known as 87.81.230.195} 212.95.237.92 (talk) 13:06, 27 July 2015 (UTC)[reply]

Deleuze & Guattari

How exactly did they co-write Anti-Oedipus and A Thousand Plateaus? Did they sit down together and thrash out every sentence between them? Did one of them write certain parts alone and the other redrafted it? Etc. --Viennese Waltz 09:41, 27 July 2015 (UTC)[reply]

Biographical research....

Any experts on Mid 19th century hymanls? see- https://en.wikisource.org/wiki/Talk:A_Comprehensive_Index_of_Names_of_Original_Authors_and_Translators_of_Psalms_and_Hymns

There are some entries that are surnames only, or in some case INITALS only.

It would be nice to expand them, response here or at wikisource. ShakespeareFan00 (talk) 13:00, 27 July 2015 (UTC)[reply]